You are on page 1of 291

www.FirstRanker.com www.FirstRanker.

com

1. The thickness of endometrium at the time


of implantation is :
a) 3 – 4 mm

b) 20 – 30 mm

c) 15 –20 mm

d) 30 – 40 mm

Correct Answer - A
Ans. is None/ a i.e. 3-4 mm
om
"7he Endometrium is in the secretory phase corresponding to 20 -
21 days of cycle" at the time of implantation.
r.c

"After ovulation, the endometrium now demonstrates a combined


ke

reaction to estrogen and progesterone activity. Most impressive is


an

that total endometrial height is fixed at roughly its preovulatory


tR

extent (5 -6 mm) despite continued availability of estrogen."


irs

Reading the above text it is clear that endometrium is - 5 - 6 mm


.F

thick at the time of implantation, which is not given in the option. Still
w

if you have to mark one answer option 'a' i.e. 3 - 4 mm being closest
w
w

could be right.
Extra Edge
Implantation occurs 7 - 9 days after ovulation°.
In human, the blastocyst burrows in the uterine cavity till whole of it
lies within the thickness of endometrium. This is called as interstitial
implantation.
After implantation of the embryo the uterine endometrium is called
the decidua".
Decidua basalis - The part of decidua where the placenta is to be
formed.
Decidua Capsularis - The part of the decidua that separates the
embryo from the uterine lumen. Decidua Parietalis - The part of the

www.FirstRanker.com
www.FirstRanker.com www.FirstRanker.com

embryo from the uterine lumen. Decidua Parietalis - The part of the
decidua lining rest of the uterine cavity.
At the end of pregnancy, the decidua is shed off along with placenta
and membranes.

om
r.c
ke
an
tR
irs
.F
w
w
w

www.FirstRanker.com
www.FirstRanker.com www.FirstRanker.com

2. Zygote with zona pellucida reaches uterine


cavity by :
a) 2 days

b) 4 days

c) 5 days

d) 6 days

Correct Answer - B
4 days
om
r.c
ke
an
tR
irs
.F
w
w
w

www.FirstRanker.com
www.FirstRanker.com www.FirstRanker.com

3. Ligamentum terea is formed after ':


a) Obliteration of the umbilical vein

b) Obliteration of the ductus venous

c) Obliteration of the ductus arteriosus

d) Obliteration of the hypogastric artery

Correct Answer - A
Obliteration of the umbilical vein om
r.c
ke
an
tR
irs
.F
w
w
w

www.FirstRanker.com
www.FirstRanker.com www.FirstRanker.com

4. The role of human placental lactogen is :


a) Stimulate milk production

b) Fetal breast development

c) Growth of fetus

d) Endocrine regulation

Correct Answer - C
Growth of fetus
Functions :
a) Provide fetal nutrition by antagonizing the action of insulin in
om

maternal circulation, breakdown of fats and proteins and transport of


r.c

fatty acids and amino acids from maternal to feta circulation.


ke

b) Potent angiogenic, helps develop fetal vasculature.


an

c) Promotes growth of breast for lactation.


tR
irs
.F
w
w
w

www.FirstRanker.com
www.FirstRanker.com www.FirstRanker.com

5. Commonest cause for puerperal sepsis is :


a) Streptococci

b) Anaerobes

c) Gonococci

d) Staphylococci

Correct Answer - A
Streptococci om
r.c
ke
an
tR
irs
.F
w
w
w

www.FirstRanker.com
www.FirstRanker.com www.FirstRanker.com

6. The best method for inducing mid trimester


abortion is :
a) Injection of Hypertonic Saline

b) Ethacrydine

c) Prostaglandins

d) D and C

Correct Answer - C
Prostaglandins
om
Mid trimester termination of pregnancy
A) Medical methods
r.c

i) Prostaglandins : misoprostol (PGE 1) with or without Mife


ke

pristone, gemeprost (PGE1), dinoprostone (PGE2), carboprost


an

(PGE2)
tR

ii) Oxytocin
irs
.F
w
w
w

www.FirstRanker.com
www.FirstRanker.com www.FirstRanker.com

7. Most common site of Implantation of tubal


pregnancy is :
a) Interstitial portion of fallopian tube

b) Isthmus

c) Ampulla

d) Infundibulum

Correct Answer - C
Ampulla
om
Maximum propensity to rupture —> Isthmic ectopic pregnancy.
Minimum propensity to rupture -3 Ampullary ectopic pregnancy.
r.c
ke
an
tR
irs
.F
w
w
w

www.FirstRanker.com
www.FirstRanker.com www.FirstRanker.com

8. During which gestational age is peak serum HCG levels attained ?


a) 7-9 weeks

b) 11-13 weeks

c) 20 weeks

d) 25 weeks

Correct Answer - A
HCG rises progressively from conception. Levels double on the average, every 30.9 hours
until values reach 6500 mIU/ml (6,500 IU/L) at approximately the eighth week after the last
menstrual period (LMP).
After that the rate of rise becomes individualized, peaking between the 60th and 70th
om

day (9 to 10 weeks) LMP.


HCG decreases slightly between the 12th and 16th week post LMP, and then remains
r.c

constant until birth.


ke
an
tR
irs
.F
w
w
w

www.FirstRanker.com
www.FirstRanker.com www.FirstRanker.com

9. The uterine artery is a branch of which of the following?


a) Left common iliac artery

b) Internal iliac artery

c) Internal pudendal artery

d) Ovarian artery

Correct Answer - B
Uterine arteries arise from internal iliac artery or hypogastric artery.
It runs downwards and forwards along the lateral pelvic wall until the base of the broad
ligament.
om

Note: During pregnancy the uterine arteries hypertrophy and their course is straightened.
r.c

Ref: Comp Textbook of Obstetrics and Gynecology, Sadhana Gupta, 2011, Page 18
ke
an
tR
irs
.F
w
w
w

www.FirstRanker.com
www.FirstRanker.com www.FirstRanker.com

10.

om
r.c
ke
an
tR
irs
.F
w
w
w

www.FirstRanker.com
www.FirstRanker.com www.FirstRanker.com

In which part of fallopian tube ectopic pregnancy will have longest survival?

a) Isthmus

b) Ampulla

c) Cornua

d) Interstitium

Correct Answer - D
Ans. D. Interstitium
If the implantation occurs in the antimesenteric border in the
ampulla, the pregnancy may continue a little longer time.
om
Earliest interruption occurs in the isthmial implantation and
pregnancy may continue upto 3-4 months in interstitial
r.c

implantation.
ke

Also know:
an

Isthmic rupture usually occurs at 6-8 weeks, the ampullary one at 8-


tR

12 weeks and the interstitial one at about 4 months.


irs

Ref: Textbook of Obstetrics by DC Dutta, 6th edition, Page 181.


.F
w
w
w

www.FirstRanker.com
www.FirstRanker.com www.FirstRanker.com

11. What would be the type of presentation when the engaging diameter is
mentovertical?

a) Face

b) Brow

c) Vertex

d) Breech

Correct Answer - B
In brow presentation, the engaging diameter is mentovertical (14cm). Brow is the
rarest variety of cephalic presentation where the presenting part is the brow and the attitude
of the of the head is short of that degree of extension necessary to produce face
presentation.
om

In face presentation, the presenting part is face, attitude of the fetus shows complete
r.c

flexion of the limbs with extension of the spine. The commonest position is left mento
anterior. The engaging diameter of the head is submento bregmatic 9.5cm in fully
ke

extended head or submento vertical 11.5cm in partially extended head.


an
tR

Ref: Textbook of Obstetrics By D.C Dutta, 6th edn, page 392-3


irs
.F
w
w
w

www.FirstRanker.com
www.FirstRanker.com www.FirstRanker.com

12. Which of the following statements regarding fetal circulation is correct?


a) The liver and heart of the fetus receive blood with very high
oxygen saturation

b) PO2 of fetal blood leaving the placenta is slightly greater than


maternal mixed venous PO2

c) The presence of fetal hemoglobin shifts the oxyhemoglobin


dissociation to the right

d) The foramen ovale closes during the third trimester unless the
fetus has an atrial septal defect
om

Correct Answer - A
r.c

Since the liver is supplied by umbilical venous blood from the placenta, and the heart
and head receive blood before it has mixed with significant amounts of desaturated
ke

blood, these important organs receive blood that is relatively high in saturated
an

oxyhemoglobin.
tR

The high rate of blood flow at the placenta and the significant resistance of the placenta to
irs

diffusion of oxygen result in blood in the umbilical vein that has a lower PO2 (30 mm Hg)
.F

than the maternal mixed venous blood.


w
w

However, the left shift in fetal oxyhemoglobin concentration and the Bohr effect both act to
w

increase the transport of oxygen to fetal tissues.

A number of significant differences in circulating patterns are present in the fetus.

The foramen ovale remains open until after birth and a significant portion of inferior
vena cava flow is shunted through it to the left.
1/3 of blood enters directly from right atrium to left atrium (bypassing right ventricle) through
foramen oval

The major portion of right ventricular output is shunted through the ductus arteriosus to the
aorta, not the lungs.

The net effect of these shunts in the presence of high fetal pulmonary vascular resistance is
very low fetal pulmonary blood flow.

www.FirstRanker.com
www.FirstRanker.com www.FirstRanker.com

At birth, these patterns normally are quickly changed to ex-utero patterns with high
pulmonary perfusion.

Ref: Barrett K.E., Barman S.M., Boitano S., Brooks H.L. (2012). Chapter 33. Circulation
through Special Regions. In K.E. Barrett, S.M. Barman, S. Boitano, H.L. Brooks (Eds),
Ganong's Review of Medical Physiology, 24e.

om
r.c
ke
an
tR
irs
.F
w
w
w

www.FirstRanker.com
www.FirstRanker.com www.FirstRanker.com

13. Most common cause of secondary PPH is


:
a) Uterine inertia

b) Retained placenta

c) Episiotomy

d) Cervical tear

Correct Answer - B
Retained placenta
om
Secondary PPH
Bleeding usually occurs between 8th to 14th day of delivery.
r.c

Causes of late PPH :


ke

Retained bits of cotyledon or membranes (M.C.)


an

Infection and separation of slough over a deep cervico - vaginal


tR

laceration.
irs

Endometritis and subinvolution of the placental site.


.F
w
w
w

www.FirstRanker.com
www.FirstRanker.com www.FirstRanker.com

14. The major contribution of the amniotic


fluid after 20 weeks of gestation :
a) Ultrafiltrate and maternal plasma

b) Fetal urine

c) Fetal lung fluid

d) Fetal skin

Correct Answer - B
Ans. B. Fetal urine
om
There are two primary sources of amniotic fluid: fetal urine and the
lung liquid.
r.c

After about 20 weeks, fetal urine makes up most of the fluid.


ke
an
tR
irs
.F
w
w
w

www.FirstRanker.com
www.FirstRanker.com www.FirstRanker.com

15.

om
r.c
ke
an
tR
irs
.F
w
w
w

www.FirstRanker.com
www.FirstRanker.com www.FirstRanker.com

Incidence of scar rupture in a subsequent


preg​nancy in case of Lower Segment
Cesarean Section (LSCS) is:
a) 2%

b) 4%

c) 5%

d) 8%

Correct Answer - A
om

2%
The risk of lower segment scar - rupture is low (0.2-1.5%) and even if
r.c
ke

it does occur, maternal death is much less and the perinatal


mortality is about 1 in 8.
an
tR
irs
.F
w
w
w

www.FirstRanker.com
www.FirstRanker.com www.FirstRanker.com

16. The characteristics of caput succedaneum


include all of the following except :
a) Crosses midline

b) Crosses the suture line

c) It does not disappear within 2-3 days

d) It is a diffuse edematous swelling of the soft tis​sues of the scalp

Correct Answer - C
It does not disappear within 2-3 days
om
Location- subcutaneous plane
Clinical features-
r.c

Soften gradually and disappear within 2-3 days


ke

Diffuse crosses suture line ill defined margin.


an

Not associated with prolonged jaundice


tR
irs
.F
w
w
w

www.FirstRanker.com
www.FirstRanker.com www.FirstRanker.com

17. Hydrocephalus is best detected


antenately by :
a) X-ray abdomen

b) Amniocentesis

c) Clinical examination

d) Ultrasonography

Correct Answer - D
Ans. is d i.e. Ultrasonography
om
Hydrocephalus is a condition in which there is an abnormal increase
in cerebrospinal fluid within the ventricular and subarachnoid spaces
r.c

of brain.
ke

"The prenatal diagnosis of hydrocephalus is usually made by


an

demonstration of a dilated ventricular system in an ultrasound


tR

examination."
irs

Earliest and most accurate sonographic sign of hydrocephalus –


.F

Enlarged lateral ventricles.°


w

The lateral ventricle is measured at the level of atrium.


w
w

Normal transverse diameter of atrium is 7 mm ± 1 mm. (It remains


constant during the second and third trimester)
When diameter of atrium is >10 mm it is called as Ventriculomegaly /
Hydrocephalus.
Other signs of hydrocephalus on USG :
Dangling choroid plexuses.
Thinning out of cerebral cortex.
Extra Edge :
Friends, the terms ventriculomegaly and hydrocephalus are often
used interchangeably but have slightly different meanings.
* Ventriculomegaly : The condition in which lateral ventricles of the

www.FirstRanker.com
www.FirstRanker.com www.FirstRanker.com

brain are filled with excessive fluid and enlarge.


* Hydrocephalus : There is ventriculomegaly along with an increase
in the head circumference.
Normal fetal head circumference at term ranges between 32 and 38
cms.
With hydrocephalus, the circumference exceeds 50 cms.
Ventriculomegaly can also be caused by° :
* Spina bifida°
* Chromosomal abnormalities°
* Congenital infections like cytomegalovirus. toxoplasmosis, syphilis
and influenza.°
Dandy walker syndrome includes Hydrocephalus + Posterior fossa
cyst°+ Defect in cerebellar vermis.

om
r.c
ke
an
tR
irs
.F
w
w
w

www.FirstRanker.com
www.FirstRanker.com www.FirstRanker.com

18. Subpubic angle is :


a) <65°

b) 65-75°

c) 85°

d) 110-120°

Correct Answer - C
85° om
r.c
ke
an
tR
irs
.F
w
w
w

www.FirstRanker.com
www.FirstRanker.com www.FirstRanker.com

19. Rule of Hasse is used to determine :


a) The age of fetus

b) Height of an adult

c) Race of a person

d) Identification

Correct Answer - A
Ans. is a i.e. The age of fetus
Hasse's rule is employed in calculating the age of fetus by its
length
om

During the first five months of pregnancy, the length in cms is square
r.c

of the age in months i.e. Length in cms = (Age in months)?


ke

Age in months = Qlength in cms


an

During second five months of pregnancy, length in cms divided by 5


tR

is the age in months. Length in ems


irs

Age in months 5
.F
w
w
w

www.FirstRanker.com
www.FirstRanker.com www.FirstRanker.com

20. Banana and lemon sign seen in which


fetal nomalies :
a) Neural tube defect

b) Hydrops fetalis

c) Twins

d) IUD

Correct Answer - A
Ans. is a i.e. Neural tube defect
om
Signs of Spina bifida on Ultrasound
Small biparietal diameter.
r.c

Ventriculomegaly.
ke

Frontal bone scalloping or the so called lemon sign.


an

Elongation and downward displacement of the cerebellum-the so


tR

called banana sign.


irs

Effacement or obliteration of the cisterna megna.


.F
w
w
w

www.FirstRanker.com
www.FirstRanker.com www.FirstRanker.com

21. Indication of Acyclovir in pregnancy :


a) Disseminated herpes

b) Chicken-pox in first trimester

c) Prophylaxis in recurrent herpes

d) All of the above

Correct Answer - D
All of the above om
r.c
ke
an
tR
irs
.F
w
w
w

www.FirstRanker.com
www.FirstRanker.com www.FirstRanker.com

22. Lovset manoeuvre is used in delivery of :


a) Head

b) Breech

c) Foot

d) Arms

Correct Answer - D
Arms
Lovset's maneuver
Principle : Because of the curved birth canal, when the anterior
om

shoulder remains above the symphysis pubis, the posterior shoulder


r.c

will be below the sacral promontory. If the fetal trunk is rotated


ke

keeping the back anterior and maintaining a downward traction, the


an

posterior shoulder will appear below the symphysis pubis.


tR
irs
.F
w
w
w

www.FirstRanker.com
www.FirstRanker.com www.FirstRanker.com

23. Macrosomia is/are associated with:


a) Gestational diabetes mellitus

b) Maternal obesity

c) Hypothyroidism

d) A & B

Correct Answer - D
Ans. is a and b i.e. Gestational diabetes and Maternal obesity
Macrosomia is the term used to describe a large fetus.
The recommended definition is fetal (neonatal) weight exceeding
om

two standard deviations or above 90th centile for the appropriate


r.c

normal population.
ke

According to ACOG : birth weight of > 4500gm is called as


an

macrosomia.
tR

In Indian context Birth weight of > 4000 gm is called as macrosomia.


irs
.F
w
w
w

www.FirstRanker.com
www.FirstRanker.com www.FirstRanker.com

24. Drug which is contraindicated before 2nd


stage of labor is:
March 2009
a) Mifepristone

b) Oxytocin

c) Misoprostol

d) Ergometrine

Correct Answer - D
om

Ans. D: Ergometrine
r.c

Drugs used for medical method of induction of labour are:


ke

Mifepristone
an

Oxytocin
tR

Misoprostol (Prostaglandins El)


irs

Ergometrine is contraindicated in pregnancy, 1st stage of labour, 2nd


.F

stage of labour before crowning of the head and in breech delivery


w

prior to crowning.
w
w

www.FirstRanker.com
www.FirstRanker.com www.FirstRanker.com

25. Nuchal translucency in USG can be


detected at_____weeks of gestation.
a) 11-13 weeks

b) 18-20 weeks

c) 8-10 weeks

d) 20-22 weeks

Correct Answer - A
Ans. A. 11-13 weeks
om
Nuchal translucency is the normal fluid-filled subcutaneous space
identified at the back of the fetal neck during the late first trimester
r.c

and early second trimester (11.3-13.6 weeks).


ke
an
tR
irs
.F
w
w
w

www.FirstRanker.com
www.FirstRanker.com www.FirstRanker.com

26. First line of treatment of mastitis in a


lactating mother is-
a) Dicloxacillin

b) Cefazolin

c) Ceftriaxone

d) Ampicillin

Correct Answer - A
Ans. A. Dicloxacillin
om
The beta lactamase-resistant penicillins have been recommended in
the treatment of mastitis. These include cloxacillin, dicloxacillin, or
r.c

flucloxacillin.
ke

Because penicillins are acidic, they are poorly concentrated in


an

human milk, which is also acid.


tR

Therefore, cloxacillin and its congeners tend to treat cellulitis well,


irs

but they are less effective in eradicating adenitis, the most likely
.F

precursor of breast abscess.


w

When patients are allergic to penicillins, cephalexin or clindamycin


w
w

may be the alternative to erythromycin.


Combination like co-amoxiclav should be avoided because of fear of
inducing MRSA

www.FirstRanker.com
www.FirstRanker.com www.FirstRanker.com

27. Patient with recurrent abortion diagnosed


to have antiphospholipid syndrome. What
will be the treatment?
a) Aspirin only

b) Aspirin + Low molecular weight Heparin

c) Aspirin + Low molecular weight Heparin + Prednisolone

d) No Treatment

Correct Answer - B
om

Ans. B. Aspirin + Low molecular weight Heparin


r.c

The recommended treatment for women with recurrent pregnancy


loss associated with antiphospholipid syndrome includes combined
ke

Aspirin and Heparin therapy.


an

In pregnant SLE patients with Antiphospholipid antibodies and prior


tR

fetal loss, treatment with heparin (standard or low –molecular-


irs

weight) plus low dose aspirin has been shown in prospective


.F

controlled trials to increase significantly the proportion of live births.


w
w

Combined aspirin and heparin therapy has proven effectiveness and


w

is the preferred treatment for women with recurrent pregnancy loss


associated with antiphospholipid syndrome.

www.FirstRanker.com
www.FirstRanker.com www.FirstRanker.com

28. Human placenta is ?


a) Discoid

b) Hemochorial

c) Deciduate

d) All the above

Correct Answer - D
Ans. is 'd' i.e., All the above
The human placenta is :
Discoid, because of its shape.
om

Hemochorial, because of direct contact of the chorion with the


r.c

maternal blood andDeciduate, because some maternal tissue is


ke

shed at parturition.
an
tR
irs
.F
w
w
w

www.FirstRanker.com
www.FirstRanker.com www.FirstRanker.com

29. Twin pregnancy of the same age and sex


rules out ?
a) Superfetation

b) Maternal twins

c) Superfecundation

d) None of the above

Correct Answer - A
Ans. is 'a' i.e., Superfetation
om
Superfetation
It is the simultaneous occurrence of more than one stage of
r.c

developing offspring in the same animal.


ke

In mammals, it manifests as the formation of an embryo from a


an

different estrous cycle while another embryo or fetus is already


tR

present in the uterus.


irs
.F
w
w
w

www.FirstRanker.com
www.FirstRanker.com www.FirstRanker.com

30. All are true about uteroplacental


circulation except
a) Blood in the intervillous space is completely replaced 3 - 4
times per minute

b) The villi depend on the maternal blood for their nutrition

c) A mature placenta has 150 ml of blood in the villi system and


350 ml of blood in the intervillous space

d) Intervillous blood flow at term is 500 - 600 ml per minute


om
Correct Answer - C
Answer- C. A mature placenta has 150 ml of blood in the villi
r.c

system and 350 ml of blood in the intervillous space


ke

A mature placenta has a volume of about 500 ml of blood; 350 ml


an

being occupied in the villi system and 150 ml lying in the intervillous
tR

space.
irs

Intervillous blood flow at term is around 500 - 600 ml per minute.


.F

The blood in the intervilllous space is completely replaced about 3 -


w

4 times per minute.


w
w

The villi depend on maternal circulation for nutrition, thus it is


possible for the chorionic villi to survive for a varying period even
after the fetus is dead.

www.FirstRanker.com
www.FirstRanker.com www.FirstRanker.com

31. Uterus is receptive for implantation for


how many days after fertilization -
a) 6 days

b) 12 days

c) 6 weeks

d) 12 weeks

Correct Answer - D
Answer- D. 12 weeks
om
Superfetation is the fertilization of two ova released in two different
menstrual cycles. The nidation and development of one fetus over
r.c

another fetus is theoretically possible until the decidual space is


ke

obliterated by 12 weeks of pregnancy.


an
tR
irs
.F
w
w
w

www.FirstRanker.com
www.FirstRanker.com www.FirstRanker.com

32. Number of stem villi at term in human


placenta is
a) 60

b) 120

c) 240

d) 480

Correct Answer - A
Answer- A. 60
om
Functional subunit is called a lobule which is derived from a tertiary
stem villi. About 60 stem villi persist in human placenta. Thus each
r.c

cotyledon (total 15 - 29) contains 3 - 4 major stem villi. The villi are
ke

the functional unit of the placenta. The total villi surface, for
an

exchange, approximately varies between 10 to 14 square metres.


tR

The fetal capillary system within the villi is almost 50 km long.


irs
.F
w
w
w

www.FirstRanker.com
www.FirstRanker.com www.FirstRanker.com

33. All are of value in modified Bishop score


except
a) Dilatation

b) Effacement

c) Cervical length

d) Consistency

Correct Answer - B
Answer- B. Effacement
om
r.c
ke
an
tR
irs
.F
w
w
w

www.FirstRanker.com
www.FirstRanker.com www.FirstRanker.com

34. G2P1L1 female with 1 : 4 anti D titres at 28


weeks gestation, management is
a) MCA Doppler

b) Caesarean section

c) Induction of labour

d) Amniocentesis

Correct Answer - A
Answer- A. MCA Doppler
om
If Indirect Coomb's test is positive in an antenatal patient with Rh
negative blood group, and antibody titres >1 : 16 or Ab level > 10
r.c

IU/ml
ke

1. Serial MCA Doppler, every 1 - 2 weeks from 20 weeks


an

2. Serial ultrasonography every 2 - 3 weeks from 20 weeks


tR
irs
.F
w
w
w

www.FirstRanker.com
www.FirstRanker.com www.FirstRanker.com

35. Patient with NTD, dose of folic acid in next


pregnancy
a) 0.5 mg

b) 1 mg

c) 2 mg

d) 4 mg

Correct Answer - D
Answer- D. 4 mg
om
Folic acid supplementation 4 mg daily 1 month before conception to
about 12 weeks of pregnancy.
r.c
ke
an
tR
irs
.F
w
w
w

www.FirstRanker.com
www.FirstRanker.com www.FirstRanker.com

36. Kallmans syndrome is associated with all


of the following except
a) Amenorrhea

b) Excess stimulation of the HPO axis

c) Genetic mutation

d) Anosmia

Correct Answer - B
Answer- B. Excess stimulation of the HPO axis
om
When congenital GnRH deficiency is associated with anosmia or
hyposmia (an absent or grossly impaired sense of smell), the
r.c

disorder is known as Kallmann's syndrome.


ke

Two genetic mutations associated with Kallman's Syndrome :


an

1. KAL gene - X - linked inheritance (Xp22.3) encoding anosmin - 1.


tR

2. Gene encoding FGFR1 (Fibroblast growth factor - 1 receptor) -


irs

autosomal dominant form.


.F

Anosmin - 1 is a neural adhesion molecule that promotes migration


w

of GnRH neurons and olfactory neurons, from the olfactory placode


w
w

into the hypothalamus during embryonic development.

www.FirstRanker.com
www.FirstRanker.com www.FirstRanker.com

37. Hot flushes are experienced as a result


of
a) Increased noradrenaline

b) Decreased estrogen

c) Increased noradrenaline and decreased estrogen

d) Increased noradrenaline and estrogen

Correct Answer - C
Answer- C. Increased noradrenaline and decreased estrogen
om
Hot flushes are caused by noradrenaline, which disturbs the
thermoregulatory system. Oestrogen deficiency reduces
r.c

hypothalamic endorphins, which release more norepinephrine and


ke

serotonin. This leads to inappropriate heat loss mechanism.


an

Other causes that can be associated with the symptom of hot


tR

flushes include: thyroid disease, epilepsy, pheochromocytoma,


irs

carcinoid syndromes, autoimmune disorders, mast cell disorders,


.F

insulinoma, pancreatic tumours and even leukemias.


w
w
w

www.FirstRanker.com
www.FirstRanker.com www.FirstRanker.com

38. Cephalic index is


a) BPD/OFD

b) BPD/HC

c) OFD/BPD

d) HC/FL

Correct Answer - A
Answer- A. BPD/OFD
Cephalic index = BPD / OFD (Biparietal diameter divided by the
occipito - frontal diameter)
om
r.c
ke
an
tR
irs
.F
w
w
w

www.FirstRanker.com
www.FirstRanker.com www.FirstRanker.com

39. If the symphysiofundal height is 40 cm


and the station of the head is at -1, weight
of the fetus is approximately
a) 3 kg

b) 3.3 kg

c) 4 kg

d) 4.3 kg

Correct Answer - D
om

Answer- D. 4.3 kg
r.c

Johnson's Formula for estimation of fetal weight :


Height of the uterus above the symphysis pubis in centimeters
ke

minus 12, if the vertex is at or above the level of ischial spines or


an

minus 11, if the vertex is below the level of ischial spines - multiplied
tR

by 155 gives the weight in grams.


irs

Solution : (40 - 12) x 155 = 4340 gms


.F
w
w
w

www.FirstRanker.com
www.FirstRanker.com www.FirstRanker.com

40. In MRKH syndrome, which among the


following is absent?
a) Vagina

b) Breast development

c) Pubic hair development

d) Testes

Correct Answer - D
Answer- D. Testes
om
MRKH syndrome is also known as MURCS syndrome (Mullerian
agenesis, Renal aplasia and cervicothoracic somite dysplasia).
r.c

MRKH (Mayer Rokitansky Kuster Hauser) syndrome has a


ke

karyotype of 46, XX. So the gonads present are ovaries in contrast


an

to Androgen Insensitivity Syndrome where testes are present.


tR

The MRKH syndrome is characterized by congenital aplasia of the


irs

uterus and the upper part (2/3) of the vagina in women showing
.F

normal development of secondary sexual characteristics and a


w

normal 46, XX karyotype.


w
w

www.FirstRanker.com
www.FirstRanker.com www.FirstRanker.com

41. What is the preferred treatment of


complete prolapse in a female with
completed family?
a) Sling surgery

b) Vaginal hysterectomy

c) Le Forte's repair

d) Pessary

Correct Answer - B
om

Answer- B. Vaginal hysterectomy


r.c

Vaginal hysterectomy is commonly performed for major degree


uterine prolapse in a female with completed family.
ke
an
tR
irs
.F
w
w
w

www.FirstRanker.com
www.FirstRanker.com www.FirstRanker.com

42. Which of the following is not a probable


sign of pregnancy
a) Jacquemier's sign

b) Dalrymple sign

c) Hegar's sign

d) Palmer's sign

Correct Answer - B
Answer- B. Dalrymple sign
om
Probable signs of pregnancy :
Jacquemier's sign/Chadwick's sign
r.c

Osiander's sign
ke

Goodell's sign
an

Piskacek's sign
tR

Hegar's sign
irs

Palmer's sign
.F

Braxtor-hicks contraction
w

External ballotment
w
w

Abdominal enlargement
Outlining the fetus

www.FirstRanker.com
www.FirstRanker.com www.FirstRanker.com

43. Polyspermy is inhibited by which ion ?


a) Ca

b) Na

c) K

d) Cl

Correct Answer - A
Ans, A. Ca
The calcium wave amplifies the local signal at the site of sperm-
oocyte interaction and distributes it throughout the oocyte cytoplasm.
om

The increase in calcium concentration is the signal that causes the


r.c

oocyte to resume cell division, initiating the completion of meiosis II


ke

and setting of the developmental programme that leads to


an

embryogenesis.
tR
irs
.F
w
w
w

www.FirstRanker.com
www.FirstRanker.com www.FirstRanker.com

44. Which of the following is present in


normal vagina?
a) Trichomonas buccalis

b) Trichomonas hominis

c) Trichomonas vaginalis

d) Trichomonas bovis

Correct Answer - C
Ans. C. Trichomonas vaginalis
om
r.c
ke
an
tR
irs
.F
w
w
w

www.FirstRanker.com
www.FirstRanker.com www.FirstRanker.com

45. Maximum amniotic fluid at ?


a) 32 weeks

b) 34 weeks

c) 36 weeks

d) 40 weeks

Correct Answer - C
Ans, C. 36 weeks om
r.c
ke
an
tR
irs
.F
w
w
w

www.FirstRanker.com
www.FirstRanker.com www.FirstRanker.com

46. At 20 weeks of gestation amniotic fluid


volume is ?
a) 200 ml

b) 400 ml

c) 600 ml

d) 800 ml

Correct Answer - B
Ans, B. 400 ml
om
r.c
ke
an
tR
irs
.F
w
w
w

www.FirstRanker.com
www.FirstRanker.com www.FirstRanker.com

47. Fertilized ovum reaches the uterus at what


day of menstrual cycle?
a) 6th

b) 14th

c) 20th

d) 25th

Correct Answer - C
Ans, C. 20th
om
Implantation of the fertilized ovum occurs in the endometrium of the
anterior or posterior wall of the body near the fundus on the & ilay of
r.c

fertilization which corresponds to the 20th day of the regular


ke

menstrual cycle.
an
tR
irs
.F
w
w
w

www.FirstRanker.com
www.FirstRanker.com www.FirstRanker.com

48.

om
r.c
ke
an
tR
irs
.F
w
w
w

www.FirstRanker.com
www.FirstRanker.com www.FirstRanker.com

Fertilization usually occurs in which part of


fallopian tube?
a) Fimbrial end

b) Ampulla

c) Interstitium

d) Isthmus

Correct Answer - B
Ans. B. Ampulla
om
Fertilization is the process of fusion of the spermatozoon with the
mature ovum.
r.c

It begins with sperm egg collision and ends with production of a


ke

mononucleated single cell called the zygote.


an

Its objectives are:


tR

1. To initiate the embryonic development of the egg and


irs

2. To restore the chromosome number of the species.


.F

Almost always, fertilization occurs in the ampullary part of the uterine


w

tube.
w
w

www.FirstRanker.com
www.FirstRanker.com www.FirstRanker.com

49. Physiologic change in leukocyte numbers


in pregnancy is
a) Neutrophilic leukocytosis

b) Lymphocytic leukocytosis

c) Neutropenia

d) Basophilic leukocytosis

Correct Answer - A
Ans. A. Neutrophilic leukocytosis
om
Physiological change observed in leukocyte numbers in pregnancy
is neutrophilic leukocytosis.
r.c

It occurs to the tune of 8000/mm3 and may upto 20,000/ mm3 in


ke

labour.
an

The increase maybe due to the rise in levels of estrogen and


tR

cortisol.
irs
.F
w
w
w

www.FirstRanker.com
www.FirstRanker.com www.FirstRanker.com

50. Weight of uterus at term is ?


a) 400-500 gm

b) 600-700gm

c) 800-900 gm

d) 900-1000 gm

Correct Answer - D
Ans. D. 900-1000 gm
The uterus in the non-pregnant state weight is about 60 gm, with a
cavity of 5 - 70 ml and measures about 7.5 cm in length.
om

At term it weighs about 900 - 1000 gm and measures 35 cm in


r.c

length.
ke
an
tR
irs
.F
w
w
w

www.FirstRanker.com
www.FirstRanker.com www.FirstRanker.com

51. During pregnancy estrogen causes which


of the following?
a) Growth of ducts of breasts

b) Growth of alveoli of breasts

c) Both a and b

d) None of the above

Correct Answer - C
Ans. C. Both a and b
om
The increased size of the breasts is evident in the early weeks of
pregnancy.
r.c

The increase in size is due to hypertrophy and proliferation of the


ke

ducts and the alveoli.


an

Estrogen is responsible for the hypertrophy and proliferation of the


tR

ducts and alveoli while progesterone is only responsible for the


irs

hypertrophy and proliferation of the alveoli.


.F
w
w
w

www.FirstRanker.com
www.FirstRanker.com www.FirstRanker.com

52. Maternal side layer of the placenta is


called ?
a) Decidua basalis layer

b) Decidua capsularis layer

c) Decidua parietalis

d) Decidua spongiosa

Correct Answer - A
Ans., A. Decidua basalis layer
om
Decidua basalis (Decidual plate) is the part of endometrium related
to embryonic pole of conceptus and forms the maternal p art of
r.c

placenta.
ke
an
tR
irs
.F
w
w
w

www.FirstRanker.com
www.FirstRanker.com www.FirstRanker.com

53. Which of the following is not a


physiological change of pregnancy in
urinary bladder?
a) Edematous mucosa

b) Increased frequency at 14 weeks

c) Stress incontinence

d) Pressure on bladder in late pregnancy

Correct Answer - B
om

Ans.B. Increased frequency at 14 weeks


r.c

Physiological changes in bladder in pregnancy


Marked congestion and hypertrophy of the muscles and elastic
ke

tissues of the bladder wall.


an

Edematous bhdder mucosa in late pregnancy especially in


tR

primigravida.
irs

Increased frequency of micturition at 6 - 8 weeks which subsides by


.F

12 weeks and reappears in late pregnancy due to pressure of the


w
w

gravid uterus on bladder.


w

Stress urinary incontinence in late pregnancy.

www.FirstRanker.com
www.FirstRanker.com www.FirstRanker.com

54. Urinary retention earliest in pregnancy is


seen at ?
a) 10 weeks

b) 18 weeks

c) 22 weeks

d) 34 weeks

Correct Answer - A
Ans, A. 10 weeks
om
Urinary retention in pregnancy is rare
It is classically described in some women with retroverted uterus,
r.c

which becomes impacted in the pelvis, usually seen earliest


ke

between 8 - 12 weeks of pregnancy and causes outflow obstruction.


an
tR
irs
.F
w
w
w

www.FirstRanker.com
www.FirstRanker.com www.FirstRanker.com

55. Beta HCG is detected earliest by which


day of conception?
a) 8 days

b) 15 days

c) 21 days

d) 30 days

Correct Answer - A
Ans, A. 8 days
om
hCG is a glycoprotein produced by the syncytiotrophoblast.
hCG-c is identical to the c subunit of LH, FSH, and TSH.
r.c

Its presence in the urine in early pregnancy is the basis of the


ke

various laboratory tests for pregnancy, and it can sometimes be


an

detected in the urine as early as 14 d after conception and in serum


tR

as early as 8-9 days,


irs
.F
w
w
w

www.FirstRanker.com
www.FirstRanker.com www.FirstRanker.com

56. Doubling time of beta HCG in early


pregnancy is ?
a) 24 hrs

b) 48 hrs

c) 72 hrs

d) 96 hrs

Correct Answer - B
Ans, B. 48 hrs
om
Beta hCG usually double about every 2 days (48 hours) during first
four weak of pregnancy.
r.c

As pregnancy progresses, doubling time becomes longer. By 6-7


ke

weeks beta hCG levels may take as long as 3.5 days to double.
an
tR
irs
.F
w
w
w

www.FirstRanker.com
www.FirstRanker.com www.FirstRanker.com

57. Tubal patency test in which phase of the


menstrual cycle?
a) Menstrual

b) Preovulatory

c) Leuteal

d) Premenstrual

Correct Answer - B
Ans, B. Preovulatory
om
The testing of tubal patency and detecting tubal pathology are done
in pre-ovulatory phase of the menstrual cycle.
r.c

If performed in the post-ovulatory period, insufflation might disturb


ke

an implanted or fertilized ovum and may also cause pelvic


an

endometriosis.
tR
irs
.F
w
w
w

www.FirstRanker.com
www.FirstRanker.com www.FirstRanker.com

58. Following physiological changes are seen


in vagina in pregnancy except ?
a) Jacquimiers sign

b) Increased length of anterior vaginal wall

c) pH acidic

d) Decreased number of navicular cells

Correct Answer - D
Ans, D. Decreased number of navicular cells
om
The vaginal walls become hypertrophied, edematous and more
vascular.
r.c

Increased blood supply to venous plexus surrounding the walls gives


ke

bluish coloration of the mucosa (Jacquemier's sign).


an

The length of the anterior vaginal wall is increased.


tR

The secretion of vagina becomes copious, thin and curdy white, due
irs

to marked exfoliated cells and bacteria.


.F

The pH becomes acidic (3.5 - 6) due to more conversion of glycogen


w

into lactic acid by lactobacillus acidophilus consequent on high


w
w

estrogen level.
The acidic pH prevents multiplication of pathogenic organisms.
There is predominance of navicular cells in cluster and plenty of
lactobacilli.

www.FirstRanker.com
www.FirstRanker.com www.FirstRanker.com

59. FHS can be usually heard by stethoscope


at ?
a) 14 weeks

b) 18 weeks

c) 22 weeks

d) 26 weeks

Correct Answer - B
Ans, B. 18 weeks
om
Fetal heart sound (FHS) is most conclusive clinical sign of
pregnancy. With an ordinary stethoscope it can be detected between
r.c

18 - 20 weeks of PregnancY.
ke
an
tR
irs
.F
w
w
w

www.FirstRanker.com
www.FirstRanker.com www.FirstRanker.com

60. What is the fetoplacental relationship at


24 weeks of gestation?
a) 3

b) 4

c) 5

d) 6

Correct Answer - A
Ans, A. 3
om
The relationship between the fetal and placental weights can be
studied by the so called fetoplacental relationship (fetal weight/
r.c

placental weight ratio).


ke

The fetoplacental relationship increases as the pregnancy advances.


an
tR
irs
.F
w
w
w

www.FirstRanker.com
www.FirstRanker.com www.FirstRanker.com

61. Down syndrome is earliest diagnosed at ?


a) 8 - 10 weeks

b) 10 - 12 weeks

c) 12 - 14 weeks

d) 14 - 16 weeks

Correct Answer - B
Ans. B. 10 - 12 weeks
Earliest diagnosis of genetic defects can be done by use of chorionic
villous sampling.
om

Chorionic villous sampling is carried out transcervically at 10 - 12


r.c

weeks and transabdominally from 10 weeks to term.


ke
an
tR
irs
.F
w
w
w

www.FirstRanker.com
www.FirstRanker.com www.FirstRanker.com

62. Chorionic villus biopsy is done earliest in


which week of gestation ?
a) 9 weeks

b) 11 weeks

c) 13 weeks

d) 15 weeks

Correct Answer - B
Ans, B. 11 weeks
om
It is carried out transcervically between 7O - 12 weeks and
transabdominally from 10 weeks to term.
r.c
ke
an
tR
irs
.F
w
w
w

www.FirstRanker.com
www.FirstRanker.com www.FirstRanker.com

63. When is folic acid started in pregnancy ?


a) 4 weeks prior to conception

b) 8 weeks prior toconception

c) 4 weeks after conception

d) 8 weeks after conception

Correct Answer - A
Ans. A. 4 weeks prior to conception
Folic acid supplementation (4mg/day) is stated 4 weeks prior to
conception and continued upto 12 weeks of pregnancy.
om

This can reduce the incidence of neural tube defects.


r.c
ke
an
tR
irs
.F
w
w
w

www.FirstRanker.com
www.FirstRanker.com www.FirstRanker.com

64. Term placenta weight to Baby weight ratio


is ?
a) 1 : 3

b) 1 : 4

c) 1 : 5

d) 1 : 6

Correct Answer - D
Ans, D. 1 : 6
om
The term Placenta
Placenta at term is a circular disc with a diameter of 15 - 20 cm.
r.c

It has thickness of 3 cm at center and thins of towards the edges.


ke

It feels spongy and weighs about 500 gm.


an

The ratio of placemat weight at term and the baby weight is 1:6.
tR

It occupies about 30% of the uterine wall.


irs
.F
w
w
w

www.FirstRanker.com
www.FirstRanker.com www.FirstRanker.com

65. First trimester diagnosis for anencephaly


is by increased?
a) Alpha feto protein in maternal serum

b) Alpha feto protein in amniotic fluid

c) Beta HCG in maternal serum

d) Beta HCG in amniotic fluid

Correct Answer - B
Ans, B. Alpha feto protein in amniotic fluid
om
In the first half of the pregnancy the diagnosis of anencephaly is
made by elevated alpha feto protein in amniotic fluid and confirmed
r.c

by sonography.
ke
an
tR
irs
.F
w
w
w

www.FirstRanker.com
www.FirstRanker.com www.FirstRanker.com

66. Which of the following is true about EDD ?


a) Less than 10% of deliveries occur on EDD

b) Less than 20% of deliveries occur on EDD

c) 80% of the deliveries occur on EDD

d) 90% of the deliveries occur on EDD

Correct Answer - A
Ans, A. Less than 10% of deliveries occur on EDD
Fewer than 5% of all the pregnancies end on the expected date of
delitery (EDD).
om

13% of the births occur preterm.


r.c

5 - 7% of the pregnancies are delivered post term.


ke

Majority of the deliveries occur within 7 days of EDD.


an
tR
irs
.F
w
w
w

www.FirstRanker.com
www.FirstRanker.com www.FirstRanker.com

67. a) Most common position of engagement


in vertex presentation?
a) LOA

b) ROA

c) LOP

d) ROP

Correct Answer - A
Ans, A. LOA
om
Vertex occupying the left anterior quadrant of the pelvis is the
commonest and is called left occipito -anterior position.
r.c
ke
an
tR
irs
.F
w
w
w

www.FirstRanker.com
www.FirstRanker.com www.FirstRanker.com

68. Which is the engaging diameter in


occipitoposterior nresentation?
a) Suboccipito frontal

b) Mento vertical

c) Submentovertical

d) Bitrochanteric

Correct Answer - A
Ans, A. Suboccipito frontal
om
r.c
ke
an
tR
irs
.F
w
w
w

www.FirstRanker.com
www.FirstRanker.com www.FirstRanker.com

69. Which is the most common presentation


in twin pregnancy?
a) Vertex - vertex

b) Vertex - breech

c) Breech - Breech

d) Vertex - Footling

Correct Answer - A
Ans. A. Vertex - vertex
om
r.c
ke
an
tR
irs
.F
w
w
w

www.FirstRanker.com
www.FirstRanker.com www.FirstRanker.com

70. Direct Occipitoposterior position is a


favourable position in which type of pelvis
?
a) Anthropoid

b) Android pelvis

c) Gynaecoid

d) Mongoloid

Correct Answer - A
om

Ans, A. Anthropoid
r.c

Direct Occipitoposterior position is a favourable position in


anthropoid type of pelvis.
ke
an
tR
irs
.F
w
w
w

www.FirstRanker.com
www.FirstRanker.com www.FirstRanker.com

71. Persistent OP position is most common in


which pelvis?
a) Android

b) Gynaecoid

c) Anthrpoid

d) Mixed

Correct Answer - A
Ans, A. Android
om
With android type of pelvis the occipitoPosterior position is common
due to funnel shape of the pelvis.
r.c
ke
an
tR
irs
.F
w
w
w

www.FirstRanker.com
www.FirstRanker.com www.FirstRanker.com

72. Contraindication for induction of labour is


all except?
a) Hypertensive disease of pregnancy

b) Heart disease of pregnancy

c) Pelvic tumor

d) Vasa previa

Correct Answer - A
Ans. A. Hypertensive disease of pregnancy
om
Hypertensive disease of pregnancy is an indication for induction of
labour. Other three are contraindications.
r.c
ke
an
tR
irs
.F
w
w
w

www.FirstRanker.com
www.FirstRanker.com www.FirstRanker.com

73. Medical management of ectopic


pregnancy has decreased success if ?
a) Gestational sac < 3cm

b) Duration of gestation < 5 weeks

c) Cardiac activity present

d) Beta HCG < 8000 IU/L

Correct Answer - C
Ans. C. Cardiac activity present
om
Medical management of an ectoPic Pregnancy is done when :
Patient is hemodynamically stable
r.c

No evidence of acute intra abdominal bleeding


ke

Ready to comply with follow up care


an

Serum beta HCG < 10,000 IU/L


tR

Absent embryonic heart activity


irs

Diameter of ectopic gestational mass less than 4 cm.


.F
w
w
w

www.FirstRanker.com
www.FirstRanker.com www.FirstRanker.com

74. All are the prognostic factors of Boer-


meisel system except -
a) Extent of adhesions

b) Thickness of tubal wall

c) Size of hydrosalpinx

d) Infecting organism

Correct Answer - D
Ans, D. Infecting organism
om
Boer-meisel system of prognostic classification for chronic
pelvic inflammatory disease includes:
r.c

Extent of adhesions
ke

Nature of adhesions flimsy or dense


an

Size of hydrosalpinx
tR

Macroscopic condition of hydrosalpinx


irs

Thickness of tubal wall


.F
w
w
w

www.FirstRanker.com
www.FirstRanker.com www.FirstRanker.com

75. Pregnant uterus will compress ureters at


?
a) Pelvic brim

b) Uterovesical junction

c) Trigone

d) Ureterovesical junction

Correct Answer - A
Ans, A. Pelvic brim
om
Ureters become atonic due to high progesterone level in pregnancy,
Dilatation of the ureter above the pelvic brim with stasis is marked
r.c

on the right side especially in the primigravidae.


ke

It is due to deoxtrorotation of the uterus pressing the right ureter


an

against the pelvic brim and abo due to pressure by right ovarian
tR

vein, which crosses the right ureter at right angle.


irs
.F
w
w
w

www.FirstRanker.com
www.FirstRanker.com www.FirstRanker.com

76. Following is given to a patient with pre


term labour exept -
a) Glucocorticoids

b) Tocolytic drugs

c) Antibiotics

d) Beta blocker

Correct Answer - D
Ans. D. Beta blocker
om
Tocolytic drugs [Note: betamimetic and not beta blocker is a
tocolytic]
r.c
ke
an
tR
irs
.F
w
w
w

www.FirstRanker.com
www.FirstRanker.com www.FirstRanker.com

77. Most common cause of tenth day post


partum bleeding?
a) Retained bits of membrane

b) Infection

c) Endometritis

d) Blood coagulopathy

Correct Answer - A
Ans. A. Retained bits of membrane
om
Postpartum hemorrhage (PPH) is defined as blood loss of more than
500 ml following birth of baby.
r.c
ke
an
tR
irs
.F
w
w
w

www.FirstRanker.com
www.FirstRanker.com www.FirstRanker.com

78. A multgravida 4 kg fetus is in labour since


15 hours and has 5cm dilation of cervix
for last 8 hours. What is the further
management of this patient ?
a) Wait and watch

b) Amniotomy

c) Injection Oxytocin

d) Caesarian section
om

Correct Answer - D
r.c

Ans. D. Caesarian section


ke
an
tR
irs
.F
w
w
w

www.FirstRanker.com
www.FirstRanker.com www.FirstRanker.com

79. The treatment of choice for bartholin cyst


is ?
a) Marsupilisation

b) Aspiration

c) Observe

d) Curettage and closure

Correct Answer - A
Ans. A. Marsupilisation
om
Bartholin's cyst
Bartholin's cyst is formed when duct of bartholins gland is blocked
r.c

either by inflammation or by inspissated secretion.


ke

It appears as a swelling on the inner side of the junction of the


an

anterior two-thirds with the posterior one-third of the labium majus.


tR

A small cyst remains asymptomatic, but a larger one bulges across


irs

the vaginal introitus and causes dyspareunia, discomfort and may


.F

get infected when it needs excision or marsupialization


w
w
w

www.FirstRanker.com
www.FirstRanker.com www.FirstRanker.com

80. Gartner's cyst are seen at ?


a) Antero lateral vaginal wall

b) Antero - lateral cervix

c) Posterolateral vaginal wall

d) Posterolateral cervix

Correct Answer - A
Ans, A,’. Antero lateral vaginal wall
Gartners duct cyst arises from the remnants of the mesotephric duct
and lies in the anterolateral aspect of vaginal wall.
om
r.c
ke
an
tR
irs
.F
w
w
w

www.FirstRanker.com
www.FirstRanker.com www.FirstRanker.com

81. Most common cause of death of baby in


vasa previa is?
a) Infection

b) Maternal exanguination

c) Fetal exanguination

d) Both b and c

Correct Answer - C
Ans, C. Fetal exanguination
om
Vasa previa
In it a leash of blood vessels happen to traverse through the
r.c

membranes overlying the internal os, in front of presenting part.


ke

Rupture of membranes involving the overlying vessels leads to


an

vaginal bleeding.
tR

As it is entirely fetal blood, this may result in fetal exanguination and


irs

even death.
.F
w
w
w

www.FirstRanker.com
www.FirstRanker.com www.FirstRanker.com

82. Sheehan syndrome is ?


a) Pitutary adenoma

b) Pitutary necrosis

c) Adrenal necrosis

d) Adrenal adenoma

Correct Answer - B
Ans., B. Pitutary necrosis
Sheehan's syndrome is anterior pituitory necrosis following severe
PPH, shock or severe infection.
om
r.c
ke
an
tR
irs
.F
w
w
w

www.FirstRanker.com
www.FirstRanker.com www.FirstRanker.com

83. Most common cause of menorrhagia in


adolescents?
a) Thyroid disorder

b) Coagulation disorders

c) Leiomyomas

d) Polyps

Correct Answer - B
Ans, B. Coagulation disorders
om
In adolescent age group, abnormal uterine bleeding results from
anovulation and coagulation defects at disproportionately higher
r.c

rates compared with older reproductive-aged women


ke

Coagulation disorders account for 20% of cases of menorrhagia in


an

adolescents.
tR
irs
.F
w
w
w

www.FirstRanker.com
www.FirstRanker.com www.FirstRanker.com

84. Not associated with endometrial


hyperplasia ?
a) PCOD

b) Glucose intolerance

c) HRT

d) Unopposed exposure to progesterone

Correct Answer - D
Ans. D. Unopposed exposure to progesterone
om
Following are the causes of endometrial hyperplasia
Follicular cysts of ovary
r.c

PCOD
ke

Granulosa and theca cell tumors of ovary


an

HRT
tR

Glucose in tolerance
irs

Unopposed exposure to estrogen (endogenous or exogenous)


.F
w
w
w

www.FirstRanker.com
www.FirstRanker.com www.FirstRanker.com

85. Following are the causes of maternal


deaths in patients with hypertensive
disorder of pregnanacy except ?
a) Cardiac failure

b) ARDS

c) Chronic renal failure

d) Cerebral hemorrhage

Correct Answer - C
om

Ans. C. Chronic renal failure


r.c

Causes of maternal deaths in cases of hypertensive disorders


of pregnancy are
ke

Cardiac failure
an

Cerebral hemorrhage
tR

ARDS
irs

Puerperal sepsis
.F

Pulmonary edema
w
w

Acute renal failure


w

Pulmonary embolism
Aspiration and/or septic pneumonia
Cardio-pulmonary arrest
Post-partum shock

www.FirstRanker.com
www.FirstRanker.com www.FirstRanker.com

86. Classical name of mid cycle abdominal


pain with vaginal bleeding is called ?
a) Endometriosis

b) Mittelschmez

c) Meteropathiahemorrhagica

d) Menometrorrhagia

Correct Answer - B
Ans. B. Mittelschmez
om
Mittelschmerz is a mid-cycle pain, not lasting more than 12-24
hours, around ovulation. Pain is located in one of the iliac fossa and
r.c

may be accompanied with vaginal bleeding.


ke
an
tR
irs
.F
w
w
w

www.FirstRanker.com
www.FirstRanker.com www.FirstRanker.com

87. Following is true about leuteoma of


pregnancy ?
a) Usually bilateral

b) It is a benign self limiting condition

c) It consists of leutenized cells

d) All the above

Correct Answer - D
Ans. D. All the above
om
Leuteoma of Pregnancy
It usually appears as bilateral, multinodular, solid masses in ovaries.
r.c

It is characterized by replacement of normal ovarian parenchyma by


ke

solid proliferation of leutenized stromal cells under influence of


an

human chorionic gonadotroPin.


tR

It is benign self-limiting condition and requires no treatment.


irs
.F
w
w
w

www.FirstRanker.com
www.FirstRanker.com www.FirstRanker.com

88. Clinical alarming sign of MgSO4 toxicity is


?
a) Loss of knee jerk

b) Loss of superficial abdominal reflexes

c) Loss of pin prick sensation

d) Loss of proprioception

Correct Answer - A
Ans. A. Loss of knee jerk
om
r.c
ke
an
tR
irs
.F
w
w
w

www.FirstRanker.com
www.FirstRanker.com www.FirstRanker.com

89. False about MgSo4 is ?


a) Not used as antihypertensive

b) Its dose is different for eclampsia and preeclampsia

c) Deep tendon reflexes is monitored for toxicity

d) It acts as a membrane stabilizer and neuroprotector

Correct Answer - B
Ans. B. Its dose is different for eclampsia and preeclampsia
Dose of magnesium sulphate for management of pre-eclampsia and
eclampsia is the same.
om
r.c
ke
an
tR
irs
.F
w
w
w

www.FirstRanker.com
www.FirstRanker.com www.FirstRanker.com

90. 18 weeks pregnancy of a lady, last two


times history of midtrimester abortion,
which was painless. What is the diagnosis
?
a) Incompetent os

b) Chromosomal abnormality

c) Bivalve uterus

d) Progesterone deficiency
om

Correct Answer - A
r.c

Ans. A. Incompetent os
ke

Most common cause of second trimester pregnancy loss is cervical


an

incompetence, in which patient presents with recurrent painless


tR

abortion.
irs
.F
w
w
w

www.FirstRanker.com
www.FirstRanker.com www.FirstRanker.com

91. Following are the features of true labour


pain ?
a) Uterine contractions at regular intervals

b) Progressive effacement and dilation of cervix

c) Formation of bag of membranes

d) All the above

Correct Answer - D
Ans. D. All the above
om
Features of true labour Pains
Uterine contractions at regular intervals
r.c

Frequency of contractions increase gradually


ke

Intensity and duration of contractions increase progressively


an

Associated with show


tR

Progressive effacement and dilation of cervix


irs

Descent of the presenting part


.F

Formation of bag of forewaters


w

Not relieved by enema and sedatives


w
w

www.FirstRanker.com
www.FirstRanker.com www.FirstRanker.com

92. HRT improves ?


a) Bone density

b) Demetia

c) Coronary artery disease

d) Endometrial cancer

Correct Answer - A
Ans, A. Bone density
Hormone replacement therapy improves bone density,
om
r.c
ke
an
tR
irs
.F
w
w
w

www.FirstRanker.com
www.FirstRanker.com www.FirstRanker.com

93. Most common risk factor for rupture of


scarred uterus is
a) Use of oxytocin in labour

b) Grand multiparity

c) Forceps application

d) Obstructed labour

Correct Answer - A
Ans, A. Use of oxytocin in labour
om
The most common cause of rupture of scarred uterus k use of high
doses of oxytocin for the augmuntation of labour.
r.c
ke
an
tR
irs
.F
w
w
w

www.FirstRanker.com
www.FirstRanker.com www.FirstRanker.com

94. Invalueable tool in the diagnosis of


chronic pelvic pain is ?
a) Endometrial biopsy

b) Ultrasound

c) Laparoscopy

d) Colposcopy

Correct Answer - C
Ans, C. Laparoscopy
om
Laproscopy is an invalueable diagnostic tool in the investigation of
chronic pelvic pain.
r.c
ke
an
tR
irs
.F
w
w
w

www.FirstRanker.com
www.FirstRanker.com www.FirstRanker.com

95. Most common site of ectopic pregnancy is


?
a) Ovary

b) Fallopian tube

c) Peritoneum

d) Cervix

Correct Answer - B
Ans. B. Fallopian tube
om
r.c
ke
an
tR
irs
.F
w
w
w

www.FirstRanker.com
www.FirstRanker.com www.FirstRanker.com

96. Least common site of ectopic pregnancy


in fallopian tubes is ?
a) Interstitium

b) Ampulla

c) Infundibulum

d) Isthumus

Correct Answer - A
Ans, A. Interstitium
om
Most common site for ectopic pregnancy fallopian tubes,
r.c
ke
an
tR
irs
.F
w
w
w

www.FirstRanker.com
www.FirstRanker.com www.FirstRanker.com

97. Criteria for Puerperal pyrexia is


temperature ?
a) 100.4 degrees F on two separate occasions

b) 101 degrees F on two separate occasions

c) 100.4 degrees F on three separate occasions

d) 101 degrees F on three separate occasions

Correct Answer - A
Ans, A. 100.4 degrees F on two separate occasions
om
Puerperal pyrexia
A rise of temperature reaching 100.4 degrees F (38 degrees C) or
r.c

more (measured orally) on two separate occasions at 24 hours apart


ke

(excluding first 24 hours) within first 10 days following delivery is


an

called puerperal pyrexia.


tR
irs
.F
w
w
w

www.FirstRanker.com
www.FirstRanker.com www.FirstRanker.com

98. Investigation of choice for diagnosis of


PID is ?
a) Laparoscopy

b) Colposcopy

c) Hysteroscopy

d) Ultrasonography

Correct Answer - A
Ans, A. Laparoscopy
om
Laparoscopy is considered the investigation of choice for the
diagnosis of pelvic inflammatory disease.
r.c
ke
an
tR
irs
.F
w
w
w

www.FirstRanker.com
www.FirstRanker.com www.FirstRanker.com

99. Which drug is preferred for the treatement


of 21 hydroxylase deficient female fetus to
prevent genital virilization?
a) Materal cortisol

b) Maternal dexamethasone

c) Maternal hydrocortisone

d) Maternal methylprednisolone

Correct Answer - B
om

Ans, b. Maternal dexamethasone


r.c

Fetus is at risk of CAH maternal dexamethasone therapy can


suppress the fetal HPA axis and prevent
ke

genital virilization in affected female fetus.


an
tR
irs
.F
w
w
w

www.FirstRanker.com
www.FirstRanker.com www.FirstRanker.com

100. Speilberg criteria is used for ?


a) Ovarian pregnancy

b) Ovarian malignancy

c) Cervical pregnancy

d) Cervical malignancy

Correct Answer - A
Ans, A. Ovarian pregnancy om
r.c
ke
an
tR
irs
.F
w
w
w

www.FirstRanker.com
www.FirstRanker.com www.FirstRanker.com

101. Main factor responsible for increased


perinatal mortality in twin pregnancy is ?
a) Prematurity

b) IUGR

c) Polyhydramnios

d) Uterine rupture

Correct Answer - A
Ans, A. Prematurity
om
r.c
ke
an
tR
irs
.F
w
w
w

www.FirstRanker.com
www.FirstRanker.com www.FirstRanker.com

102. Cryptomenorrhoea is a feature of ?


a) Vaginal atresia

b) Turner syndrome

c) Empty sella syndrome

d) Gonadal agenesis

Correct Answer - A
Ans, A. Vaginal atresia om
r.c
ke
an
tR
irs
.F
w
w
w

www.FirstRanker.com
www.FirstRanker.com www.FirstRanker.com

103. . Which of the following antiepileptic


drug is associated with causing
congenital heart disease in fetus?
a) Barbiturates

b) Valproate

c) Carbamazepine

d) Phenytoin

Correct Answer - A
om

Ans. A. Barbiturates
r.c
ke
an
tR
irs
.F
w
w
w

www.FirstRanker.com
www.FirstRanker.com www.FirstRanker.com

104. Engagement of head in labour means ?


a) Smallest horizontal plane of the presenting part has crossed the
pelvic brim

b) Greatest horizontal plane of the presenting part has crossed the


pelvic brim

c) Smallest horizontal plane of the presenting part has crossed the


pelvic outlet

d) Greatest horizontal plane of the presenting part has crossed the


pelvic outlet
om

Correct Answer - B
r.c

Ans. B. Greatest horizontal plane of the presenting part has


ke

crossed the pelvic brim


an

When the greatest horizontal plane of the presenting part has


tR

passed the plane of pelvic brim the presenting part is said to be


irs

engaged.
.F
w
w
w

www.FirstRanker.com
www.FirstRanker.com www.FirstRanker.com

105. USG of 28 weeks gestation showing


oligohydramnios is likely to be due to ?
a) Gastroinstestinal obstruction

b) Renal pathway obstruction

c) Anencephyaly

d) Neuromuscular disorder

Correct Answer - B
Ans, B. Renal pathway obstruction
om
Urinarytract obstruction is an important cause of oligohydramnios.
r.c
ke
an
tR
irs
.F
w
w
w

www.FirstRanker.com
www.FirstRanker.com www.FirstRanker.com

106. Anovulatory DUB is due to ?


a) Absence of progesterone

b) Excess of estrogen

c) Hypothalmic pitutary defect

d) High progesterone

Correct Answer - A
Ans. A. Absence of progesterone
No progesterone is produced when ovulation does not occur, and
thus proliferative endometrium persists.
om

At the tissue level, persistent proliferative endometrium is often


r.c

associated with stromal breakdown, decreased spiral arteriole


ke

density, and increased dilated and unstable venous capillaries.


an

At the cellular level, the availability of arachidonic acid is reduced,


tR

and prostaglandin production is impaired.


irs

For these reasons, bleeding associated with anovulation is thought


.F

to result from changes in endometrial vascular structure and in


w

prostaglandin concentration, and from an increased endometrial


w

responsiveness to vasodilating prostaglandins.


w

www.FirstRanker.com
www.FirstRanker.com www.FirstRanker.com

107. In carcinoma cervix surgery to retain


conception is done in which stage ?
a) 1B1

b) 1B2

c) 2A

d) 2B

Correct Answer - A
Ans. A. 1B1
om
Concepts is retained upto term only in stage I A1.
In other stages, hysterectomy is done with fetus left in situ (if it is not
r.c

viable); or delivery of fetus by classic cesarean hysterectomy


ke

followed by radiation (if fetus is viable).


an
tR
irs
.F
w
w
w

www.FirstRanker.com
www.FirstRanker.com www.FirstRanker.com

108. Bishop scoring is done for ?


a) Exchange transfusion in newborn

b) Induction of labour

c) Ventilation of Newborn

d) Gestation of Newborn

Correct Answer - B
Ans. B. Induction of labour
Bishop score, also Bishop's score, also known as cervix score is a
pre-labor scoring system to assist in predicting whether induction of
om

labor will be required.


r.c
ke
an
tR
irs
.F
w
w
w

www.FirstRanker.com
www.FirstRanker.com www.FirstRanker.com

109. Teratozoospermia refers to?


a) Absence of semen

b) Absence of sperm

c) All dead sperms in ejaculate

d) Morphologically defective sperms

Correct Answer - D
Ans. D. Morphologically defective sperms
om
r.c
ke
an
tR
irs
.F
w
w
w

www.FirstRanker.com
www.FirstRanker.com www.FirstRanker.com

110. Bispinous diameter in anatomic outlet ?


a) 10.5 cm

b) 11 cm

c) 11.5 cm

d) 12 cm

Correct Answer - B
Ans. B. 11 cm
Anatomical outlet
Antero-posterior (13 cm)
om

Distance between lower border of symphysis pubis to the tip of


r.c

coccyx.
ke

Transverse diameter (bispinous- 11 cm)


an

Distance between the inner borders of ischialtuberosities


tR

Posterior sagittal diameter (8.5 cm)


irs

Distance between sacroccygeal joint and anterior margin of anus.


.F
w
w
w

www.FirstRanker.com
www.FirstRanker.com www.FirstRanker.com

111. Most suitable type of pelvis in female ?


a) Gynaecoid

b) Android

c) Anthropoid

d) Platypelloid

Correct Answer - A
Ans. A. Gynaecoid
Gynaecoid pelvis round in shape & is most spacious.
om
r.c
ke
an
tR
irs
.F
w
w
w

www.FirstRanker.com
www.FirstRanker.com www.FirstRanker.com

112. Which pelvis is associated with DTA ?


a) Android

b) Antrhropoid

c) Platypelloid

d) Gynaecoid

Correct Answer - A
Ans. A. Android om
r.c
ke
an
tR
irs
.F
w
w
w

www.FirstRanker.com
www.FirstRanker.com www.FirstRanker.com

113. In obstructive labor most important


parameter is?
a) Diameter of pelvic inlet

b) Diameter of pelvic outlet

c) Biparietal diameter

d) Bitemporal diameter

Correct Answer - B
Ans. B. Diameter of pelvic outlet
om
The narrowest diameter for the fetus to pass through is pelvic outlet.
r.c
ke
an
tR
irs
.F
w
w
w

www.FirstRanker.com
www.FirstRanker.com www.FirstRanker.com

114. Maximum diameter that passed through


maternal pelvis ?
a) Suboccipital bregmatic

b) Biparietal

c) Suboccipital frontal

d) Occipito frontal

Correct Answer - B
Ans, B. Biparietal
om
Occipitofrontal is large diameter and deflexed head may complicate
as deep transverse arrest and obstructed labour.
r.c

In case of spacious gynaecoid and anthropoid pelvis it ends up in


ke

face to pubis delivery.


an

Mento-vertical is the largest diameter but it cannot pass through the


tR

maternal pelvis.
irs
.F
w
w
w

www.FirstRanker.com
www.FirstRanker.com www.FirstRanker.com

115. Semen examination can be done ?


a) Immediately in semi solid form

b) After liquefaction

c) Within 15-30 minutes of liquefaction

d) 11/2 - 2 hr irrespective of liquefaction

Correct Answer - D
Ans. D. 11/2 - 2 hr irrespective of liquefaction
Semen analysis
Best specimen obtained by masturbation
om

3-5 days of abstinence is best, not more than that


r.c

Specimen should be examined within 1 1/2 - 2hr


ke

Liquefaction usually completes within 15-20 min and if delayed it


an

indicates prostatic dysfunction. Thus during semen analysis timing of


tR

liquefaction is important but timing of analysis is not set according to


irs

liquefaction.
.F
w
w
w

www.FirstRanker.com
www.FirstRanker.com www.FirstRanker.com

116. Lithotomy position increases vaginal


opening by how many cm -
a) 1cm

b) 2cm

c) 3cm

d) 4cm

Correct Answer - B
Ans. B. 2cm
om
In dorsal lithotomy position, the antero-posterior diameter of the
outlet may be increased to 1.5-2 cm.
r.c

Furthermore, the coccyx is pushed back while the head ilescends


ke

down to the perineum.


an
tR
irs
.F
w
w
w

www.FirstRanker.com
www.FirstRanker.com www.FirstRanker.com

117. Endometrial biopsy is usually done at ?


a) Just before menstruation

b) 10-12 days after menstruation

c) Just after menstruation

d) At the time of ovulation

Correct Answer - A
Ans. A. Just before menstruation om
r.c
ke
an
tR
irs
.F
w
w
w

www.FirstRanker.com
www.FirstRanker.com www.FirstRanker.com

118. Oligomenorrhoea means ?


a) Cycle < 20 days

b) Cycle more than 45 days

c) Cycle more than 28 days

d) Cycle more than 35 days

Correct Answer - D
Ans. D. Cycle more than 35 daysnee om
r.c
ke
an
tR
irs
.F
w
w
w

www.FirstRanker.com
www.FirstRanker.com www.FirstRanker.com

119. Magnification obtained by colposcopy is


?
a) 1-2 times

b) 5-6 times

c) 15-25 times

d) 10-20 times

Correct Answer - D
Ans. D. 10-20 times
om
Colposcope provides a magnification of 10-20 times.
Colpomicroscopeprovides a magnification 100-300 times.
r.c
ke
an
tR
irs
.F
w
w
w

www.FirstRanker.com
www.FirstRanker.com www.FirstRanker.com

120. On which day LH & FSH should be


measured ?
a) 1-3rd day

b) 7th day

c) 14th day

d) 10th day

Correct Answer - A
Ans. A. 1-3rd day
om

Level of the hormone FSH, LH and estradiol arc measured on day 3


r.c

in infertility workup.
An elevated FSH level on day 3 is an indication of poor ovarian
ke

reserve.
an

Hormonal assessment FSH/LH developingfollicle producesestrogen


tR

which signals the hypothalamus to increase or reduce the amount of


irs

FSH produced by the pituitary gland. When good follicles are not
.F

developingestrogen levels are lower and more FSH is produced.


w
w

This leak to higher level of FSH on day 3.


w

Day 3 FSH fertility test b not the only parameter used to assess
ovarian reserve.

www.FirstRanker.com
www.FirstRanker.com www.FirstRanker.com

121. Best parameter for estimation of fetal


age by ultrasound in 3rd trimester is ?
a) Femur length

b) BPD

c) Abdominal circuference

d) Intraoccular distanc

Correct Answer - A
Ans. A. Femur length
om
Best parameters for estimation of fetal age by ultrasound
1st trimester – Crown Rump length (CRL)
r.c

2nd trimester - Corrected biparietal diameter (cBPD) or head


ke

circumference (HC)
an

3d trimester - Head circumference and femur length


tR

Overall - Crown rump length


irs
.F
w
w
w

www.FirstRanker.com
www.FirstRanker.com www.FirstRanker.com

122. Commonest variety of compound


presentation is ?
a) Head with hand

b) Head with foot

c) Head with both foot

d) Head, hand & foot

Correct Answer - A
Ans. A. Head with hand
om
When a cePhalic presentation is complicated by the presence of a
hand or a foot or both alongside the head or presence of one orboth
r.c

hands by the side of the breech it is called compound presentation.


ke

Commonest variety of compound presentation - Head with hand


an

Rarest variety of compound presentation - Head with hand and foot


tR

both.
irs
.F
w
w
w

www.FirstRanker.com
www.FirstRanker.com www.FirstRanker.com

123. Hegar sign?


a) Uterine contraction

b) Quickening

c) Bluish discoloration of vagina

d) Softening of isthmus

Correct Answer - D
Ans. D. Softening of isthmus
Hegars sign
Present in 2/3 of cases.
om

Demonstrated in 6-10 week


r.c

Signs is based on the fad that:


ke

1. Upper part of the body of the uterus is enlarged by growingfetus.


an

2. Lower part of the body is empty and extremely soft (just above the
tR

cervix)
irs

3. Cervixis comparatively firm.


.F

Because of variation in consistent on bimanual examination (two


w

fingers in anterior fornix and the abdominal fingers behind the


w

uterus) the abdominal and vaginal fingers seem to oppose below


w

thebody of the uterus.

www.FirstRanker.com
www.FirstRanker.com www.FirstRanker.com

124. Pregnant women going for long journey


& prolonged sitting is associated with
danger of ?
a) Thromboembolism

b) Seat belt compression

c) Preterm labor

d) Bleeding

Correct Answer - A
om

Ans. A. Thromboembolism
r.c

Prolonged sitting in car or aeroplane is avoided due to venous stasis


or thromboembolism.
ke
an
tR
irs
.F
w
w
w

www.FirstRanker.com
www.FirstRanker.com www.FirstRanker.com

125. Head of baby is removed in breech


delivery by which maneuver ?
a) Lovsets maneover

b) Pinards maneover

c) Prague

d) Burn Marshall method

Correct Answer - D
Ans. D. Burn Marshall method
om
r.c
ke
an
tR
irs
.F
w
w
w

www.FirstRanker.com
www.FirstRanker.com www.FirstRanker.com

126. Prague manuever is used for ?


a) After coming head in breech

b) Deep transverse assest

c) Extraction of extended arms

d) External cephalic version

Correct Answer - A
Ans. A. After coming head in breech
Sometimes the head rotatesposteriorly so, that the face is behind
the pubis.
om

Delivery in this position is difficult and 'Prague maneuver' may be


r.c

tried.
ke
an
tR
irs
.F
w
w
w

www.FirstRanker.com
www.FirstRanker.com www.FirstRanker.com

127.

om
r.c
ke
an
tR
irs
.F
w
w
w

www.FirstRanker.com
www.FirstRanker.com www.FirstRanker.com

Trial of labour in previous casearian section


can be done in ?
a) Placenta previa type III

b) Previous two classical casearian section

c) Suspected CPD

d) Previous caesarian section with adequate pelvis

Correct Answer - D
Ans. D. Previous caesarian section with adequate pelvis
om
Adequate pelvis is an indication oftrial oflabour.
Other three options are contraindications.
r.c
ke
an
tR
irs
.F
w
w
w

www.FirstRanker.com
www.FirstRanker.com www.FirstRanker.com

128. Hematoma during labour is not due to ?


a) Improper haemostasis

b) Extension of cervical laceration

c) Rupture of paravaginal venous plexus

d) Obliteration of dead space while suturing vaginal wall

Correct Answer - D
Ans. D. Obliteration of dead space while suturing vaginal wall
Failure of obliteration of dead space causes hematoma (not
obliteration of dead space).
om
r.c
ke
an
tR
irs
.F
w
w
w

www.FirstRanker.com
www.FirstRanker.com www.FirstRanker.com

129. Treatment of jaundice in third trimester ?


a) Termination of pregnancy

b) Termination at 42 weeks

c) Termination at 38 weeks

d) Wait for spontaeous labour

Correct Answer - C
Ans, C. Termination at 38 weeks
In patients of intrahepatic cholestasis of pregnancy there is
increased perinatal mortality.
om

Hence fetal surveillance is done with biweekly NST.


r.c

Conventional antepartum testing does not predict fetal mortality as


ke

there is sudden death in cholestasis due to acute hypoxia.


an

Hence delivery is recommended at 37-38 weeks.


tR

In those patients with jaundice with S bilirubin > 1.8 mg%,


irs

termination ofpregnancy should be done at 36 weeks.


.F
w
w
w

www.FirstRanker.com
www.FirstRanker.com www.FirstRanker.com

130. Rate of turnover of amniotic fluid is ?


a) 500 cc/h

b) 1L/hr

c) 1500 cc/h

d) 2L/h

Correct Answer - A
Ans. A. 500 cc/h
Specific gravity of Amniotic fluid: 1.008 to 1.010
Osmolality: 250 mosm/L
om

Completely replaced in 3 hours.


r.c

Rate of amniotic fluid turn over is 500cc/hr.


ke
an
tR
irs
.F
w
w
w

www.FirstRanker.com
www.FirstRanker.com www.FirstRanker.com

131. Maximum amount of amiotic fluid is seen


at how many weeks ?
a) 16

b) 30

c) 12

d) 38-40

Correct Answer - D
Ans. D. 38-40
om
r.c
ke
an
tR
irs
.F
w
w
w

www.FirstRanker.com
www.FirstRanker.com www.FirstRanker.com

132. Uterus post pregnancy becomes a pelvic


organ in?
a) 4 weeks

b) 6 weeks

c) 12 weeks

d) 2 week

Correct Answer - D
Ans, D. 2 week
om
By the end of 2 weeks - uterus is a pelvic organ.
By the md of 6 week - uterus returns almost to its normal size (pre
r.c

pregnant size).
ke
an
tR
irs
.F
w
w
w

www.FirstRanker.com
www.FirstRanker.com www.FirstRanker.com

133. Assisted head delivery is done in ?


a) Brow presentation

b) Face presentation

c) Persistent occipito posterior position

d) Twin presentation 1522. % of

Correct Answer - C
Ans. C. Persistent occipito posterior position
Occipito posterior positions
"In practice about 5-1O% of women admitted in labourwith cephalic
om

presentations present with occipito-posterior presentations.


r.c

Given time and patience, many of these will rotate and get corrected
ke

to occipito-anterior position and deliver normally.


an
tR
irs
.F
w
w
w

www.FirstRanker.com
www.FirstRanker.com www.FirstRanker.com

134. % of women delivering on their EDD is ?


a) 25%

b) 50%

c) 4%

d) 15%

Correct Answer - C
Ans. C. 4%
Based on the Noegelesfomulalabour starts approximately on: -
Expected date – 4%
om
r.c
ke
an
tR
irs
.F
w
w
w

www.FirstRanker.com
www.FirstRanker.com www.FirstRanker.com

135. Inner cell mass differentiates into ?


a) Chorion

b) Trophoectoderm

c) Embryo

d) All of the above

Correct Answer - C
ANs. C. Embryo
Blastocyst enlarges & the zonapellucida undergoes lysis , this is
called zona hatching.
om

The cells on the outer side become trohoectoderm which


r.c

differentiates into chorion.


ke

The cells on the inner side form inner cell mass which differentiates
an

into embrvo.
tR
irs
.F
w
w
w

www.FirstRanker.com
www.FirstRanker.com www.FirstRanker.com

136. Embryo is called "fetus" after how many


weeks post fertilization/conception ?
a) 6

b) 8

c) 10

d) 12

Correct Answer - B
Ans. B. 8
om
Embryonic period begins at 3rd week following ovulation/fertilization
& extends upto 8 weeks post conception (10 weeks from LMP).
r.c

Fetal period begins after 8 weeks post conception (10 weeks from
ke

LMP) & ends in delivery.


an
tR
irs
.F
w
w
w

www.FirstRanker.com
www.FirstRanker.com www.FirstRanker.com

137. Utero-placental circulation is


established days after fertilization ?
a) 5

b) 10

c) 15

d) 20

Correct Answer - B
Ans. B. 10
om
The uteroPlacental circulation is established 9-10 days after
fertilization.
r.c

Fetoplacental circulation is established 2l days post fertilization.


ke
an
tR
irs
.F
w
w
w

www.FirstRanker.com
www.FirstRanker.com www.FirstRanker.com

138. Pregnancy is contraindicated in all of the


following except -
a) Primary Pulmonary Hypertension

b) Eisenmenger's syndrome

c) Marfan's with aortic root dilation

d) WPW syndrome

Correct Answer - D
Ans, D. WPW syndrome
om
r.c
ke
an
tR
irs
.F
w
w
w

www.FirstRanker.com
www.FirstRanker.com www.FirstRanker.com

139. Which heart disease has the worst


prognosis/maximum mortality in
pregnancy?
a) MS

b) AS

c) PDA

d) Eisenmenger's syndrome

Correct Answer - D
om

Ans. D. Eisenmenger's syndrome


r.c
ke
an
tR
irs
.F
w
w
w

www.FirstRanker.com
www.FirstRanker.com www.FirstRanker.com

140. Ovarian cycle can be correlated with all


except ?
a) Endometrial sampling

b) Vaginal cytology

c) Blood hormonal levels

d) Estrous cycle

Correct Answer - D
Ans. D. Estrous cycle
om
Estrous cycle does not occur in human beings.
r.c
ke
an
tR
irs
.F
w
w
w

www.FirstRanker.com
www.FirstRanker.com www.FirstRanker.com

141. Endometrial biopsy to detect ovulation is


done on which day of the menstual cycle
?
a) Day 8-9

b) Day 13-15

c) Day 21-23

d) Day 3-5

Correct Answer - C
om

Ans. C. Day 21-23


r.c

Endometrial Biopsy : Rarely done now a days for the purpose of


iletecting ovulation.
ke
an
tR
irs
.F
w
w
w

www.FirstRanker.com
www.FirstRanker.com www.FirstRanker.com

142. The uterine blood flow at term is -


a) 50 mL/min

b) 100-150 mL/min

c) 350-375 mL/min

d) 500-750 mL/min

Correct Answer - D
Ans. D. 500-750 mL/min
Uteroplacental blood flow increases progressively during pregnancy
and ranges from 5N)-800 mL/minat term.
om
r.c
ke
an
tR
irs
.F
w
w
w

www.FirstRanker.com
www.FirstRanker.com www.FirstRanker.com

143. Godell's sign is ?


a) Dusky hue of the vestibule

b) Softening of the cervix

c) Increased pulsations felt through the lateral fornices

d) Regular and rhythmic contractions during bimanual examination

Correct Answer - B
Ans. B. Softening of the cervix om
r.c
ke
an
tR
irs
.F
w
w
w

www.FirstRanker.com
www.FirstRanker.com www.FirstRanker.com

144. In fetus, insulin production begin at


weeks of gestation -
a) 4-6

b) 8-12

c) 14-18

d) 24-28

Correct Answer - B
Ans. B. 8-12
om
Reaching 8 to 10 weeks into development, the pancreas starts
producing insulin, glucagon, somatostatin, and pancreatic
r.c

polypeptide.
ke
an
tR
irs
.F
w
w
w

www.FirstRanker.com
www.FirstRanker.com www.FirstRanker.com

145. Limb bud appear at what weeks of


gestation?
a) 3

b) 4

c) 6

d) 9

Correct Answer - B
Ans. B. 4
om
The upper extremity is first discretely visible as a bulge or limb bud
that develops on the ventrolateral wall of the embryo on day 26 (4-
r.c

mm crown-to-rump length).
ke
an
tR
irs
.F
w
w
w

www.FirstRanker.com
www.FirstRanker.com www.FirstRanker.com

146. Features of non severe/mild pre


ecampsia are all ecxept​ -
a) Diastolic BP <100 mm Hg

b) Systolic BP< 160 mm Hg

c) Mild IUGR

d) No premonitory symptoms

Correct Answer - C
Ans., C. Mild IUGR
om
r.c
ke
an
tR
irs
.F
w
w
w

www.FirstRanker.com
www.FirstRanker.com www.FirstRanker.com

147. FERNING is due to ?


a) Estogen & sodium chloride

b) Progesterone & sodium chloride

c) HCG

d) All of the above

Correct Answer - A
Ans, A, Estogen & sodium chloride
Cervical mucus is relatively rich in sodium chloride when estrogen
(but not progesterone) is being produced.
om
r.c
ke
an
tR
irs
.F
w
w
w

www.FirstRanker.com
www.FirstRanker.com www.FirstRanker.com

148. Second wave of trophoblastic invasion


occurs at weeks of gestation?
a) 8-11

b) 10-12

c) 12-15

d) 16-20

Correct Answer - C
Ans, C. 12-15
om
The timing of the development of the uteroplacental vessels has
been described in waves, or stages, over the course of gestation.
r.c

The first wave occurs before 12 weeks post-fertilization and consists


ke

ofinvasion and modification of the spiral arteries of the decidua.


an

Between 12 and 16 weeks pos-fertilization, the second wore occurs.


tR

This involves invasion of the intramyometrial parts of the spiral


irs

arteries, converting narrow lumen, muscular spiral arteries into


.F

dilated, low-resistance uteroplacental vessels.


w
w
w

www.FirstRanker.com
www.FirstRanker.com www.FirstRanker.com

149. Abstinence period before semen


analysis is ?
a) 1-2 days

b) 3-5 days

c) 5-7 days

d) 7-9 days

Correct Answer - B
Ans. B. 3-5 days
om
The ideal specimen for examination is after 3-5 days of abstinence.
r.c
ke
an
tR
irs
.F
w
w
w

www.FirstRanker.com
www.FirstRanker.com www.FirstRanker.com

150. Engaging diameter in face presentation


is -
a) Suboccipitobregmatic

b) Mentovertical

c) Submentobregmatic

d) Occipitofrontal

Correct Answer - C
ANs, C. Submentobregmatic
om
r.c
ke
an
tR
irs
.F
w
w
w

www.FirstRanker.com
www.FirstRanker.com www.FirstRanker.com

151. Least likely to cause dysmenorrhea -


a) Endometriosis

b) Adenomyosis

c) Uterine polyp

d) Cervical polyp

Correct Answer - D
Ans, D. Cervical polyp om
r.c
ke
an
tR
irs
.F
w
w
w

www.FirstRanker.com
www.FirstRanker.com www.FirstRanker.com

152. LH surge is due to ?


a) Progesterone

b) Estrogen

c) AMH

d) All of the above

Correct Answer - B
Ans, B. Estrogen
3-4 days before the ovulation, estrogen level crosses a certain limit
(threshold level).
om
r.c
ke
an
tR
irs
.F
w
w
w

www.FirstRanker.com
www.FirstRanker.com www.FirstRanker.com

153. Menopause is defined as ?


a) Presence of hot flushes

b) Cessation of menses for 1 year

c) Cessation of menses for 6 months

d) Cessation of menses for 2 years

Correct Answer - B
Ans, B. Cessation of menses for 1 year
Menopause is defined as the permanent cessation of menses for 7
year and is physiologically correlated with the decline in estrogen
om

secretion resulting from the loss offollicular/ovarian function.


r.c
ke
an
tR
irs
.F
w
w
w

www.FirstRanker.com
www.FirstRanker.com www.FirstRanker.com

154. Symptoms of menopause are all except ?


a) Hot flushes

b) Night sweats

c) Decrease libido

d) Intermittent hypotension

Correct Answer - D
Ans, D. Intermittent hypotension
Hot flushes
The classic symPtom associated with estrogen deficiency is the hot
om

flash, also known as hot flush


r.c

This symptom is described as 'recurrent, transient periods of


ke

flushing, sweating and a sensation ofheat, often accompanied by


an

palpations, feeling of anxiety and sometimes followed by chills".


tR
irs
.F
w
w
w

www.FirstRanker.com
www.FirstRanker.com www.FirstRanker.com

155. MENOPAUSE is diagnosed by ?


a) Estradiol <20 pg/ml

b) Progesterone <40 ng/dl

c) FSH>40 IU/L

d) LH > 20 IU/L

Correct Answer - C
Ans, C. FSH>40 IU/L
As per the American Association of clinical Endocrinologists, the
diagnosis of menopause is confirmed by FSH levels >40 IU/L.
om
r.c
ke
an
tR
irs
.F
w
w
w

www.FirstRanker.com
www.FirstRanker.com www.FirstRanker.com

156. The velocity of sperm is ?


a) 1-2 cm/hr

b) 2-4 cm/min

c) 1-4 mm/min

d) 1-4 mm/hr

Correct Answer - C
Ans. C. 1-4 mm/min
The velocity of a sperm in fluid medium is usually 1-4 mm/min.
This allows the sperm to move towards an ovum in order to fertilize
om

it.
r.c
ke
an
tR
irs
.F
w
w
w

www.FirstRanker.com
www.FirstRanker.com www.FirstRanker.com

157. After ejaculation semen liquefies in ?


a) 10 minutes

b) 30 minutes

c) 75 minutes

d) 120 minutes

Correct Answer - B
Ans. B. 30 minutes
Following ejaculation, the semen forms a gel which provides
protection for the sperm from the acidic environment of the vagina.
om

The gelisliquefied within 20-30 minute by enzymes from the prostate


r.c

gland.
ke
an
tR
irs
.F
w
w
w

www.FirstRanker.com
www.FirstRanker.com www.FirstRanker.com

158. During pregnancy, true statement about


CVS is ?
a) Cardiac output decreases

b) Right axis deviation

c) Increase in left ventricular end diastolic diameter

d) All of the above

Correct Answer - C
Ans, C. Increase in left ventricular end diastolic diameter
om
2D echo: Increase in left ventricular end diastolic diameter.
Increase in left and right atrial diameters.
r.c
ke
an
tR
irs
.F
w
w
w

www.FirstRanker.com
www.FirstRanker.com www.FirstRanker.com

159. MC site of implantation is ?


a) Fallopian tube amullary part

b) Fallopian tube isthmus

c) Fundus of uterus

d) Cornu of uterus

Correct Answer - C
Ans, C. Fundus of uterus
Implantation occurs in the endometrium on the anterior or posterior
wall of the body near the fundus on the sixth day following
om

fertilization (corresponding to the 20th day of the menstrual cycle).


r.c
ke
an
tR
irs
.F
w
w
w

www.FirstRanker.com
www.FirstRanker.com www.FirstRanker.com

160. Alpha subunit of hCG is similar to ?


a) FSH

b) LH

c) TSH

d) All of the above

Correct Answer - D
Ans, D. All of the above
Human chorionic Gonadotropin (hcG) isstructurallyrelated to three
other glycoproteinhormones LH, FSH, and TSH.
om
r.c
ke
an
tR
irs
.F
w
w
w

www.FirstRanker.com
www.FirstRanker.com www.FirstRanker.com

161. Placental hormone with highest


carbohydrate content is -
a) HCG

b) Human pregnancy specific beta glycoprotein

c) HPL

d) Relaxin

Correct Answer - A
Ans, A. HCG
om
HCGhas the highest carbohydrate content of any human hormone-
30%.
r.c
ke
an
tR
irs
.F
w
w
w

www.FirstRanker.com
www.FirstRanker.com www.FirstRanker.com

162. The pelvic inlet usually is considered to


be contracted if its shortest
anteroposterior diameter is less than -
a) 12 cm

b) 10 cm

c) 8 cm

d) 14 cm

Correct Answer - B
om

Ans. B. 10 cm
r.c

The pelvic inlet usually is considered to be contracted if its shortest


anteroposteriordiameter is less than10 cmor if the greatest
ke

transverse diameter isless than12 cm


an
tR
irs
.F
w
w
w

www.FirstRanker.com
www.FirstRanker.com www.FirstRanker.com

163. Azoospermia with normal FSH would


indicate ?
a) Hypothalamic failure

b) Testicular failure

c) Obstruction of vas defrens

d) All of the above

Correct Answer - C
Ans, C, Obstruction of vas defrens
om
r.c
ke
an
tR
irs
.F
w
w
w

www.FirstRanker.com
www.FirstRanker.com www.FirstRanker.com

164. Causes of male infertility ?


a) Idiopathic

b) Varicocele

c) Yq 11 micro deletion

d) All of the above

Correct Answer - D
Ans, D. All of the above om
r.c
ke
an
tR
irs
.F
w
w
w

www.FirstRanker.com
www.FirstRanker.com www.FirstRanker.com

165. Fetal thyroid gland is able to synthesize


hormones by weeks of gestation ?
a) 6-7

b) 7-8

c) 10-12

d) 12-14

Correct Answer - C
Ans, C. 10-12
om
Fetal thyroidglandisabletosynthesizehormonesby 10-12
weeksofgestation.
r.c
ke
an
tR
irs
.F
w
w
w

www.FirstRanker.com
www.FirstRanker.com www.FirstRanker.com

166. Poor prognosis in first trimester USG is


?
a) No fetal pole at 5 weeks

b) No cardiac activity at 5 weeks

c) No gestational sac at 4 weeks

d) No cardiac activity at 8 weeks of gestation

Correct Answer - D
Ans, D. No cardiac activity at 8 weeks of gestation
om
Absence of embryo with heartbeat 22 weeks after a scan that
showed a gestational sac without a yolk sac
r.c
ke
an
tR
irs
.F
w
w
w

www.FirstRanker.com
www.FirstRanker.com www.FirstRanker.com

167. All are true about post partum


depression except -
a) Symptoms resolve in 10-12 days

b) Affects both sexes

c) SSRIs are effective

d) None of the above

Correct Answer - A
Ans, A. Symptoms resolve in 10-12 days
om
r.c
ke
an
tR
irs
.F
w
w
w

www.FirstRanker.com
www.FirstRanker.com www.FirstRanker.com

168. Nerve injured in McRoberts maneuver is


?
a) Lumbosacral trunk

b) Obturator nerve

c) Femoral nerve

d) Pudendal nerve

Correct Answer - C
Ans, C. Femoral nerve
om
McRoberts maneuver is to be done in cases of shoulder dystocia.
When the maternal thighs are markedly flexed and abducted,
r.c

pressure from the overlying inguinal iigament may lead to femoral


ke

nerve injury.
an
tR
irs
.F
w
w
w

www.FirstRanker.com
www.FirstRanker.com www.FirstRanker.com

169. Most conclusive clinical sign of


pregnancy is ?
a) Uterine enlargement

b) Cervical softening

c) Amenorrhea

d) Fetal heat sound auscultation

Correct Answer - D
Ans. D. Fetal heat sound auscultation
om
FHS auscultation is the most conclusive clinical sign of pregnancy.
r.c
ke
an
tR
irs
.F
w
w
w

www.FirstRanker.com
www.FirstRanker.com www.FirstRanker.com

170. CRL when cardiac activity can be


detected earliest by TVS -
a) 1-4mm

b) 1 cm

c) 6-7mm

d) 2-4 cm

Correct Answer - A
Ans, A. 1-4mm
om
Fetal heart beat can be detected as early as just under 6 weeks
gestation on good quality, high frequency transvaginalultrasound, as
r.c

a crown rump length (CRL) of as little as 1-2 mm.


ke
an
tR
irs
.F
w
w
w

www.FirstRanker.com
www.FirstRanker.com www.FirstRanker.com

171. Kamla, 30 years old, P2L2 with 3.2 x 4.1


cm fibroid uterus, complains of
menorrhagia and is on symptomatic
treatment since 6 months. The patient
refuses surgery. Next line of
management is ?
a) GnRH analogs

b) Danazol

c) Myomectomy

d) Uterine artery embolization


om
r.c
ke
an

Correct Answer - D
Ans, D. Uterine artery embolization
tR

UAE can be used as a therapy for symptomatic patients who refuse


irs

or want to avoid surgery.


.F
w

After embolization, there is 60-65% decrease in size of fibroids over


w

a period of 6-9 months, and so the patient's symPtoms may


w

decrease or disaPPear.
If the patient is still symptomatic after 1 year, then surgery should be
considered.

www.FirstRanker.com
www.FirstRanker.com www.FirstRanker.com

172. Living ligature of the uterus is ?


a) Endometrium

b) Middle layer of myometrium

c) Inner layer of myometrium

d) Parametrium

Correct Answer - B
Ans, B. Middle layer of myometrium
Middle crisscross fibres act as living ligature during involution of the
uterus and prevent blood loss.
om
r.c
ke
an
tR
irs
.F
w
w
w

www.FirstRanker.com
www.FirstRanker.com www.FirstRanker.com

173. In partograms recommended by 'WHO'


the distance between the alert and action
lines is ?
a) 1 hour

b) 2 hours

c) 4 hours

d) 5 hours

Correct Answer - C
om

Ans. C. 4 hours
r.c

The concept of alert line' and ‘action line’ was introduced by Philpott
and Castle in 1972.
ke

The action line can be placed at 2 – 4 hours interval to the right and
an

parallel to alert line.


tR

In partograms recommended by'WH0'the distancebetween the alert


irs

and action lines is 4 hours.


.F
w
w
w

www.FirstRanker.com
www.FirstRanker.com www.FirstRanker.com

174. Surgical excision of corpus luteum


before weeks of gestation, results in
miscarriage
a) 6-7

b) 9-11

c) 11-12

d) 12-14

Correct Answer - A
om

Ans, A, 6-7
r.c

Surgical excision of corpus luteum (luteoctomy) before 7 weeks of


gestation, uniformly precipitated an abrupt decrease in serum
ke

progesterone concentration followed by miscarriage


an
tR
irs
.F
w
w
w

www.FirstRanker.com
www.FirstRanker.com www.FirstRanker.com

175. Factors responsible for development of


OHSS include ?
a) Histamine

b) Cytokines

c) Vascular Endothelial Growth Factor

d) All of the above

Correct Answer - D
Ans, D. All of the above
om
VEGF is considered to be the most important.
r.c
ke
an
tR
irs
.F
w
w
w

www.FirstRanker.com
www.FirstRanker.com www.FirstRanker.com

176. MVA syringe is used for ?


a) First trimester MTP

b) 2nd trimester MTP

c) Vacuum delivery

d) All of the above

Correct Answer - A
Ans, A, First trimester MTP
Manual vacuum aspiration (MvA) is a safe and effective method of
abortion that involves evacuation of the uterine contents by the use
om

of a hand-held plastic aspirator.


r.c

It is appropriate for treatment of incomplete abortion for uterine sizes


ke

up to 12 weeks from the last menstrual period (including


an

miscarriage, spontaneous abortion and removal of retained products


tR

from an induced abortion), first-trimester MTP and endometrial


irs

biopsy.
.F
w
w
w

www.FirstRanker.com
www.FirstRanker.com www.FirstRanker.com

177. Diihrssen incision is taken on -


a) Fallopian tube

b) Ovary

c) Incompletely dilated cervix

d) Fully dilated cervix

Correct Answer - C
Ans, C, Incompletely dilated cervix
Occasionalln especially with small preterm fetuses, the incompletely
dilated cervix will not allow vaginal delivery of the after-coming head
om

ofthe breech.
r.c

In such cases, Duhrssen incisions are usually necessary (cut the


ke

cervix at l0 and 2 o'clock positions).


an
tR
irs
.F
w
w
w

www.FirstRanker.com
www.FirstRanker.com www.FirstRanker.com

178. A 27-year-old female with placenta previa


had severe bleeding. What is the most
likely outcome post delivery?
a) Galactorrhea

b) Diabetes

c) Absence of menstrual cycle

d) Cushing syndrome

Correct Answer - C
om

Ans. c. Absence of menstrual cycle


r.c

Sheehan syndrome, also known as postpartum hypopituitarism or


postpartum pituitary necrosis, is hypopituitarism caused by necrosis
ke

due to blood loss and hypovolemic shock during and after childbirth.
an

Most common initial symptoms of Sheehan syndrome are


tR

agalactorrhea (absence of lactation) under difficulties with lactation.


irs

Many women also report amenorrhea or oligomenorrhea after


.F

delivery.
w
w
w

www.FirstRanker.com
www.FirstRanker.com www.FirstRanker.com

179. Woman has 100 ml blood loss every 30


days. This is called as ?
a) Menorrhagia

b) Polymenorrhea

c) Hypomenorrhea

d) Normal menses

Correct Answer - A
Ans. A. Menorrhagia
om
Normal blood loss during menses is around 35 ml (20-80 ml)
Blood loss more than 80 ml is menorrhagia
r.c
ke
an
tR
irs
.F
w
w
w

www.FirstRanker.com
www.FirstRanker.com www.FirstRanker.com

180. Menometrorrhagia is ?
a) Heavy periods

b) Intermenstrual bleeding

c) Heavy & irregular bleeding

d) Uterine bleeding occurring at regular intervals of less than 21


days

Correct Answer - C
Ans. C. Heavy & irregular bleeding
om
r.c
ke
an
tR
irs
.F
w
w
w

www.FirstRanker.com
www.FirstRanker.com www.FirstRanker.com

181. Prevalence of breech presentation at full


term is ?
a) 10%

b) 6-7%

c) 3-4%

d) 1-2%

Correct Answer - C
Ans. C. 3-4%
om
r.c
ke
an
tR
irs
.F
w
w
w

www.FirstRanker.com
www.FirstRanker.com www.FirstRanker.com

182. The shortest conjugate is ?


a) True conjugate

b) Obstetric conjugate

c) Diagonal conjugate

d) Anatomical conjugate

Correct Answer - B
Ans. B. Obstetric conjugate
Obstetric conjugate (10 cm)
Distance between midpoint of sacral Promontory to the prominent
om

bony projection in the midline on inner surface of symphysis pubis.


r.c
ke
an
tR
irs
.F
w
w
w

www.FirstRanker.com
www.FirstRanker.com www.FirstRanker.com

183. Shortest transverse diameter is ?


a) BPD

b) Bitemporal diameter

c) Bimastoid diameter

d) All are equal in length

Correct Answer - C
Ans. C. Bimastoid diameter
Bimastoid diameter = 7.5 cm
Occipitofrontal diameter → 11.5 cm
om
r.c
ke
an
tR
irs
.F
w
w
w

www.FirstRanker.com
www.FirstRanker.com www.FirstRanker.com

184. With reference to fetal heart rate, a


nonstress test is considered reactive
when?
a) Two fetal heart rate accelerations are noted in 20 minutes

b) One fetal heart rate acceleration is noted in 20 minutes

c) Two fetal heart rate accelerations are noted in 10 minutes

d) Three fetal heart rate accelerations are noted in 30 minutes

Correct Answer - A
om

Ans. A. Two fetal heart rate accelerations are noted in 20


r.c

minutes
Reactive (Reassuring) NST
ke

Two or more accelerations of > 15 beats/minute above the baseline,


an

lasting for > 75 seconds arc present in 20-4O minutes observation


tR

period.
irs
.F
w
w
w

www.FirstRanker.com
www.FirstRanker.com www.FirstRanker.com

185. Variable deceleration is seen in ?


a) Head compression

b) Uteroplacental insufficiency

c) Cord compression

d) None of the above

Correct Answer - C
Ans, C. Cord compression
Variable decelerations are due to cord compression
(oligohydramnios in labor)
om
r.c
ke
an
tR
irs
.F
w
w
w

www.FirstRanker.com
www.FirstRanker.com www.FirstRanker.com

186. Modified BIOPHYSICAL PROFILE is ?


a) NST + FETAL TONE

b) FETAL TONE + AFI

c) NST + AFI

d) NST+ FETAL TONE + AFI

Correct Answer - C
Ans, C. NST + AFI
Modified BPP = NST &AFI
BPP has 5 components
om
r.c
ke
an
tR
irs
.F
w
w
w

www.FirstRanker.com
www.FirstRanker.com www.FirstRanker.com

187. If fetus is having hypoxia, which of the


BPP parameter will be affected last ?
a) Fetal tone

b) Fetal breathing movement

c) Fetal movements

d) NST

Correct Answer - A
Ans. A. Fetal tone
om
First activity to appear, Fetal Tone at about 7.5-8.5 weeks, is also
presumably the last activity to disappear with progressively
r.c

worsening hypoxia.
ke
an
tR
irs
.F
w
w
w

www.FirstRanker.com
www.FirstRanker.com www.FirstRanker.com

188. Test used to detect genetic abnormality


in embryo,before transferring it to the
uterus in IVF is ?
a) Embryo cell biopsy

b) CVS

c) ICSI

d) All of the above

Correct Answer - A
om

Ans, A. Embryo cell biopsy


r.c

PGD, involves removing a cell from an IVF embryo to test it for a


specific genetic condition (cystic fibrosis' for example) before
ke

transferring the embryo to the uterus.


an
tR
irs
.F
w
w
w

www.FirstRanker.com
www.FirstRanker.com www.FirstRanker.com

189. In IVF, embryos are transferred back to


uterine cavity at cells stage ?
a) 2

b) 2-4

c) 4-8

d) 8-16

Correct Answer - C
Ans, C. 4-8
om
Typically embryos ate transferred at the cleavage stage (Day 2 or 3
after oocyte retrieval).
r.c

Day three embryos ate called cleavage stage embryos and have
ke

approximately 4 - 8 cells.
an
tR
irs
.F
w
w
w

www.FirstRanker.com
www.FirstRanker.com www.FirstRanker.com

190. Azoospermic patient can be a father of a


child, by which of the following?
a) IUI

b) ZIFT

c) ICSI

d) Not possible & counsel regarding adoption

Correct Answer - C
Ans. C. ICSI
om
PESA= percutaneous epididymalsperm aspiration
MESA= microscopic epididymal sperm aspiration
r.c

TESA= testicular sperm asPiration


ke

TESE= testicular sperm extraction (testicular biopsy)


an
tR
irs
.F
w
w
w

www.FirstRanker.com
www.FirstRanker.com www.FirstRanker.com

191. A primigravida with 36 weeks of


pregnancy is in labor with 3 cm dilatation
and minimal uterine contraction. On
rupture of membranes, fresh bleeding is
noted with late fetal deceleration up to 50
beats/min. The patient was taken for
LSCS but fetus could no be saved. No
abruptio or placenta previa was seen.
The likely diagnosis is ?
a) Placenta previa

b) Revealed abruptio
om

c) Circumvallate placenta
r.c
ke
an

d) Vasa previa
tR
irs
.F
w

Correct Answer - D
w

Ans. D. Vasa previa


w

Vasa previa ( l:2500) is a rare condition in which fetal blood vessels


are in front of the presenting part and cross the cervix.
The condition has a high fetal mortality rate (50-95%). This is
attributed to rapid fetal exsanguination, resulting from the vessels
tearing when the cervix dilates, membrane ruPture.

www.FirstRanker.com
www.FirstRanker.com www.FirstRanker.com

192. All are components of Active


Management of the Third Stage of Labor
except ?
a) Uterotonic agent within 1 minute of birth

b) Massage of uterus before control cord traction

c) Control cord traction

d) None of the above

Correct Answer - B
om

Ans. B. Massage of uterus before control cord traction


r.c

Administer anuterotonic drug at the delivery of the anterior shoulder


or afterwards, within one minute of the baby's birth.
ke

Before performing AMTSL, gently palpate the woman's abdomen to


an

rule out the presence of another baby. At this point, do not massage
tR

the uterus.
irs

Perform controlled cord traction


.F

Massage the uterus immediately


w
w
w

www.FirstRanker.com
www.FirstRanker.com www.FirstRanker.com

193. With which of the following events, the


feto maternal haemorrhage risk is the
least ?
a) Amniocentesis

b) Cordocentrsis

c) Chorionic villus sampling

d) Abruption

Correct Answer - D
om

Ans. D. Abruption
r.c
ke
an
tR
irs
.F
w
w
w

www.FirstRanker.com
www.FirstRanker.com www.FirstRanker.com

194. Infertility is defined as ?


a) Inability to conceive after 1 year of regular unprotected
intercourse

b) Inability to conceive after 1 year of marriage

c) Inability to conceive after 2 years of marriage

d) Inability to conceive in spite of 2 years of regular unprotected


intercourse

Correct Answer - A
ANS. A. Inability to conceive after 1 year of regular unprotected
om

intercourse
r.c

Infertility is defined as an inability to conceive in spite of 1 year of


ke

regular unprotected intercourse


an
tR
irs
.F
w
w
w

www.FirstRanker.com
www.FirstRanker.com www.FirstRanker.com

195. Cord prolapse is least likely with -


a) Transverse lie

b) Footling breech

c) Oligohydroamnios

d) Floating head

Correct Answer - C
Ans. C. Oligohydroamnios
Cord prolapse has been defined as the descent of the umbilical cord
through the cervix alongside (occult) or past the presenting part
om

(overt) in the vagina or outside the vulva in the presence of ruptured


r.c

membranes
ke
an
tR
irs
.F
w
w
w

www.FirstRanker.com
www.FirstRanker.com www.FirstRanker.com

196. Ideal time to do Glucose challenge test in


pregnancy is ?
a) 12-16 weeks

b) 20-24 weeks

c) 24-28 weeks

d) 30-34 weeks

Correct Answer - C
Ans. C. 24-28 weeks
om
O'sullivan Blood Sugar Screening Test (Glucose Challenge Test)
The ideal time to do this test is24-28 weeks of gestation (as insulin
r.c

resistance in pregnancy is maximum at 28 weeks of gestation)


ke
an
tR
irs
.F
w
w
w

www.FirstRanker.com
www.FirstRanker.com www.FirstRanker.com

197. After IUFD, when does the mother


develop DIC -
a) 48 hours

b) 1-2 weeks

c) 3-4 weeks

d) 6 weeks

Correct Answer - C
Ans. C. 3-4 weeks
om
Thromboplastin from the dead fetus can enter the maternal system
and cause DIC.
r.c

This only happens when the dead fetus is retained inside for 3-4
ke

weeks.
an
tR
irs
.F
w
w
w

www.FirstRanker.com
www.FirstRanker.com www.FirstRanker.com

198. Folic acid required in first trimester of


normal pregnancy -
a) 100 microgram

b) 400-500 microgram

c) 4 mg

d) 5 mg

Correct Answer - B
Ans, B, 400-500 microgram
om
Some NTDs are associated with a specific mutation in the
methylene tetrahydrofolatereductase gene, the adverse effects of
r.c

whichcan be largely overcome by periconceptional folic acid


ke

supplementation.
an

More than half of NTDs could be prevented with daily intake of 400
tR

microgram of folic acid throughout the periconceptional period.


irs

A woman with a prior pregnancy complicated by a neural tube defect


.F

can reduce the 23% recurrence risk by more than 70% if she takes 4
w

mg of folic acid for the month before conception and for the first
w
w

trimester of pregnancy.

www.FirstRanker.com
www.FirstRanker.com www.FirstRanker.com

199. A 30-year-old is 14 weeks pregnant. She


had two painless deliveries at 16 weeks
earlier. Next line of management is ?
a) Cervical encerclage

b) Evaluation for diabetes mellitus and thyroid disorders

c) Cervical length assessment

d) Tocolytics

Correct Answer - C
om

Ans, C. Cervical length assessment


r.c

The patient had two painless abortions at 16 weeks in the past, so


mostly it is a case of incompetent os.
ke

Next line of management in these patients is frequent cervical length


an

assessment: clinically or by USG.


tR

The patient is evaluated more frequently and if the cervix is short


irs

(less than 2.5cm) than cervical encerclage has to be done.


.F

Cervical encerclage is the surgery ofchoice for incompetent os, but


w
w

the surgery itselfcan lead to complications such as uterine


w

contractions, abortions, and PROM.


So the surgery is only to be done if it is indicated.

www.FirstRanker.com
www.FirstRanker.com www.FirstRanker.com

200. Upper two -third anterior vaginal wall


prolapse is ?
a) Cystocele

b) Urethrocele

c) Rectocele

d) Enterocele

Correct Answer - A
Ans. A. Cystocele
om
r.c
ke
an
tR
irs
.F
w
w
w

www.FirstRanker.com
www.FirstRanker.com www.FirstRanker.com

201. Earliest sign after IUFD is ?


a) Overlapping of skull bones

b) Hyperflexion of spine

c) Gas in great vessel

d) Over crowding of ribs

Correct Answer - C
Ans. C. Gas in great vessel
Robert sign (gas in great vessels) - 12hrs after death.
om
r.c
ke
an
tR
irs
.F
w
w
w

www.FirstRanker.com
www.FirstRanker.com www.FirstRanker.com

202. True about Gartners cyst is ?


a) Retention cyst in remnants of Wolffian duct

b) Arises from mullerian duct

c) Commonly arises from cervix

d) Impulse on coughing

Correct Answer - A
Ans, A. Retention cyst in remnants of Wolffian duct
om
r.c
ke
an
tR
irs
.F
w
w
w

www.FirstRanker.com
www.FirstRanker.com www.FirstRanker.com

203. If the anal spincter is injured, it is which


degree of Perineal Tear ?
a) First

b) Second

c) Third

d) Fourth

Correct Answer - C
Ans, C. Third
om
Perineal Tears are classified into four categories
First-degree tear: laceration is limited to the fourchette and
r.c

superficial perineal skin or vaginal mucosa.


ke

Second-degree tear: laceration extends beyond fourchette,


an

perinealskin and vaginal mucosa to perineal muscles and fascia, but


tR

not the anal sphincter.


irs

Third-degree tear: fourchette, perineal skin, vaginal mucosa,


.F

muscles, and anal sphincter are torn


w
w
w

www.FirstRanker.com
www.FirstRanker.com www.FirstRanker.com

204. If the rectal mucosa is injured, it is which


degree of Perineal Tear ?
a) First

b) Second

c) Third

d) Fourth

Correct Answer - D
Ans, D. Fourth
om
Rectal mucosal tear is fourth degree tear.
r.c
ke
an
tR
irs
.F
w
w
w

www.FirstRanker.com
www.FirstRanker.com www.FirstRanker.com

205. Cryptomenorrhea occurs in ?


a) Fibroids

b) PCOS

c) Imperforate hymen

d) All of the above

Correct Answer - C
Ans, C. Imperforate hymen
Congenital
Imperforate hymen: It is due to failure of disintegration of the central
om

cells of Mullerian eminence that project into urogenital sinus


r.c

TransversevaginalsePtum
ke

Atresia ofvagina, cervix.


an
tR
irs
.F
w
w
w

www.FirstRanker.com
www.FirstRanker.com www.FirstRanker.com

206. Best indicator for ovarian reserve is ?


a) AMH

b) LH/FSH ratio

c) FSH

d) Estradiol

Correct Answer - A
Ans. A. AMH
AMH blood levels are thought to reflect the size of the remaining egg
supply or “ovarian reserve"
om
r.c
ke
an
tR
irs
.F
w
w
w

www.FirstRanker.com
www.FirstRanker.com www.FirstRanker.com

207. HPL has activity similar to which


hormone ?
a) Oxytocin

b) Growth hormone

c) Insulin

d) All of the above

Correct Answer - B
Ans. B. Growth hormone
om
Human placenta lactogen (hPL) was named so, because of its
potent lactogenic & growth hormone like bioactivity as well as
r.c

immunochemical resemblance to human growth hormone'


ke
an
tR
irs
.F
w
w
w

www.FirstRanker.com
www.FirstRanker.com www.FirstRanker.com

208. Which of the following is not a soft


tissue marker of Down syndrome on
USG ?
a) Increase NT

b) Absent nasal bone

c) Exomphalos

d) Polydactyly

Correct Answer - D
om

Ans. D. Polydactyly
r.c
ke
an
tR
irs
.F
w
w
w

www.FirstRanker.com
www.FirstRanker.com www.FirstRanker.com

209. In a case of recurrent spontaneous


abortion the following investigation is
unwanted ?
a) Hysteroscopy

b) Testing for antiphospholipid antibodies

c) Testing for TORCH infections

d) Thyroid function tests

Correct Answer - C
om

Ans. C. Testing for TORCH infections


r.c

Testingfor TORCH infections is now thought to be unwarranted.


ke
an
tR
irs
.F
w
w
w

www.FirstRanker.com
www.FirstRanker.com www.FirstRanker.com

210. Dilatation & evacuation is done for all


ecxept ?
a) Inevitable abortion

b) Incomplete abortion

c) Threatened abortion

d) None of the above

Correct Answer - C
Ans. C. Threatened abortion
om
Inevitable abortion means the process ofexpulsion ofproducts
ofconception has become irreversible. The expulsion of products of
r.c

conception has not occurred but it is bound to happen and nothing


ke

can be done to stop this process.


an

When the entire products are not expelled, part of it is left inside the
tR

uterine cavity, it is called incomplete abortion.


irs
.F
w
w
w

www.FirstRanker.com
www.FirstRanker.com www.FirstRanker.com

211. WHO normal Hb value for a non pregnant


adult female is ?
a) 10 gm/di

b) 11 gm /dl

c) 12 gm/dl

d) 13 gm/dl

Correct Answer - C
Ans. C. 12 gm/dl
om
r.c
ke
an
tR
irs
.F
w
w
w

www.FirstRanker.com
www.FirstRanker.com www.FirstRanker.com

212. Outcomes of occipito posterior position?


a) Deep transverse arrest

b) Occipito sacral arrest

c) Face to pubis delivery

d) All of the above

Correct Answer - D
Ans, D. All of the above om
r.c
ke
an
tR
irs
.F
w
w
w

www.FirstRanker.com
www.FirstRanker.com www.FirstRanker.com

213. RMP can perform MTP in first trimester if


he has assisted in MTPs -
a) 5

b) 15

c) 25

d) 50

Correct Answer - C
Ans, C. 25
om
A registered medical practitioner shall have one or more of the
following experience or training in gynecology and obstetrics namely
r.c

=
ke

If he has assisted a registered medical practitioner in the


an

performance of twenty-five cases of medical termination


tR

ofpregnancy of which at least five have been performed


irs

independently,
.F
w
w
w

www.FirstRanker.com
www.FirstRanker.com www.FirstRanker.com

214. Crying of fetus in utero is called as ?


a) Vagitus uterinus

b) First cry

c) Utero vaginalis

d) Vagitus vagina

Correct Answer - A
Ans, A. Vagitus uterinus om
r.c
ke
an
tR
irs
.F
w
w
w

www.FirstRanker.com
www.FirstRanker.com www.FirstRanker.com

215. True about cephalhematoma is :


a) Crosses the suture lines

b) Always present at birth

c) Ventouse delivery is a risk factor

d) All of the above

Correct Answer - C
Ans. C. Ventouse delivery is a risk factor
The usual causes of a cephalohematoma are a prolonged second
stage of labor or instrumental delivery,particularly ventouse.
om
r.c
ke
an
tR
irs
.F
w
w
w

www.FirstRanker.com
www.FirstRanker.com www.FirstRanker.com

216. All are causes of anovulatory


amenorrhoea except?
a) PCOD

b) Hyperprolactemia

c) Gonadal dysgenesis

d) Drugs

Correct Answer - C
Ans. C. Gonadal dysgenesis
om
Causes of anovulatory amenorrhea
PCOD
r.c

Hyperprolactinaemia
ke

Weight loss, stress, exercise


an

Drugs
tR

Chest wall stimulation


irs
.F
w
w
w

www.FirstRanker.com
www.FirstRanker.com www.FirstRanker.com

217. What is false about post menopausal


state ?
a) Low LH

b) Low estrogen

c) High FSH

d) High androgen

Correct Answer - A
Ans. A. Low LH
om
Hormonal changes in post-menoPausal state
FSH level is increased
r.c

Oestrogen level is decreased & most of it is synthesised peripherally


ke

by conversion of androgen to oestrogen


an

Androgen level is slightly increased


tR
irs
.F
w
w
w

www.FirstRanker.com
www.FirstRanker.com www.FirstRanker.com

218. In a postmenopausal female, which


hormone increases?
a) FSH

b) Estrogen

c) GH

d) None of the above

Correct Answer - A
Ans, A. FSH
om
r.c
ke
an
tR
irs
.F
w
w
w

www.FirstRanker.com
www.FirstRanker.com www.FirstRanker.com

219. Role of lactobacilli in vaginal secretions


a) To maintain alkaline pH

b) To maintain acidic pH

c) Nutrition

d) None

Correct Answer - B
Ans., B. To maintain acidic pH
The importance of Doderlein's bacillus is that its presence is
associated with production of lactic acid contained in the vaigna and
om

this acidity inhibits the growth of other organisms."


r.c

Doderlein's bacillus is the only organism which will grow at the pH of


ke

4-4.5 (normal pH of vagina).


an
tR
irs
.F
w
w
w

www.FirstRanker.com
www.FirstRanker.com www.FirstRanker.com

220. Hegar sign all are true except ?


a) Bimanual palpation method

b) Difficult in obese

c) Can be done at 14 weeks

d) Present in 2/3rd of cases

Correct Answer - C
Ans. C. Can be done at 14 weeks
Hegar's sign: Present in2/3rd of cases. Demonstrated between 6-10
weeks, a little earlier in multipara. This sign is based on the 2 facts
om

(1) upper part of the body of uterus is enlarged by growing uterus


r.c

and lower part is empty and soft and cervix is comparatively film.
ke

Therefore on bimannual examination (2 fingers in the anterior fornix


an

and abdominal fingers behind the uterus), abdominal and vaginal


tR

fingers seem to appose below the body ofthe uterus.


irs
.F
w
w
w

www.FirstRanker.com
www.FirstRanker.com www.FirstRanker.com

221. Hegar sign is seen in how many weeks ?


a) 6-10 weeks

b) 10-14 weeks

c) 14-18 weeks

d) 18-22 weeks

Correct Answer - A
Ans. A. 6-10 weeks om
r.c
ke
an
tR
irs
.F
w
w
w

www.FirstRanker.com
www.FirstRanker.com www.FirstRanker.com

222. Palmer sign is related to ?


a) Contraction of uterus

b) Dusky hue of ant vaginal wall

c) Bluish discolouration of ant vaginal wall

d) Increased pulsations felt through lateral fornix

Correct Answer - A
Ans, A, Contraction of uterus
Palmer sign:-regular and rhythmic contractions can be elicited during
bimanual examination as early as 4-8 weeks.
om
r.c
ke
an
tR
irs
.F
w
w
w

www.FirstRanker.com
www.FirstRanker.com www.FirstRanker.com

223. New born can be given breast milk after


how much time following normal
delivery?
a) Half hour

b) 1 hours

c) 2 hours

d) 3 hours

Correct Answer - A
om

Ans A. Half hour


r.c

A healthy baby is put to the breast immediately or at most % to 1 hr


following normal delivery.
ke

Following caesarean sections a period of 4-6 hours may be sufficient


an

for the mother to feed her baby.


tR
irs
.F
w
w
w

www.FirstRanker.com
www.FirstRanker.com www.FirstRanker.com

224. 4 month amenorrhoea with increased


FSH, LH & decreased estrogen in a 35
yrs old?
a) Premature menopause

b) Menopause

c) Late menopause

d) Perimenopause

Correct Answer - A
om

Ans, A. Premature menopause


r.c

Premature menopause is defined as ovarian failure occurring 2 SD


in years before the mean menopausal age in a population.
ke

It is clinically defined as secondary amenorrhea for at least 3 months


an

with raised FSH level, raised FSH:LH ratio & low E2 level in a
tR

women under 40 yrs of age."


irs
.F
w
w
w

www.FirstRanker.com
www.FirstRanker.com www.FirstRanker.com

225. 35 yr old with 4 months amenorrhea with


increased FSH, decreased estrogen.
What is the diagnosis?
a) Premature ovarian failure

b) PCOD

c) Pituitary failure

d) Hypothalamic failure

Correct Answer - A
om

Ans. A. Premature ovarian failure


r.c

It is a case of premature menoPause (premature ovarian failure)'


ke
an
tR
irs
.F
w
w
w

www.FirstRanker.com
www.FirstRanker.com www.FirstRanker.com

226. Contraction stress test false is ?


a) Oxytocin not used

b) Invasive method

c) Detects fetal well being

d) Negative test is associated with good fetal outcome

Correct Answer - A
Ans, A. Oxytocin not used
Contraction stress test (CST) (syn : Oxytocin challenge test):
It is an invasive test to asses fetal respiratory well being during
om

pregnancy.
r.c

It detects alteration in FHR in response to uterine contraction


ke

induced by oxytocin indicating hypoxia.


an

Interpretations
tR

Negative test indicated good outcome


irs
.F
w
w
w

www.FirstRanker.com
www.FirstRanker.com www.FirstRanker.com

227. NST, what is seen except?


a) Variability

b) Acceleration

c) Time period

d) Oxytocin

Correct Answer - D
Ans, D. Oxytocin
The non-stress test (NST) measures fetal heart rate, which is
monitored with an external transducer for at least 20 minutes.
om

During fetal movement, tracing is observed for heart rate


r.c

acceleration.
ke

Test is positive if two or more fetal heart rate accelerations occurs in


an

20 minute period.
tR
irs
.F
w
w
w

www.FirstRanker.com
www.FirstRanker.com www.FirstRanker.com

228. Menstrual regulation effective upto ?


a) 14 days

b) 21 days

c) 4weeks

d) 6 weeks

Correct Answer - A
Ans, A. 14 days
Menstrual regulation (lnduction/Aspiration)
Aspiration of endometrial cavity within l4 days of missed period in a
om

woman with previous normal cycle.


r.c

Done as an OPD procedure.


ke

Helps to detect failed abortion, molar pregnancy or ectopic


an

Pregnancy.
tR

Contraindicated in advanced pregnancy & in presence of local pelvic


irs

inflammation.
.F
w
w
w

www.FirstRanker.com
www.FirstRanker.com www.FirstRanker.com

229. pH of vagina in pregnant woman is


usually ?
a) 4.0

b) 4.5

c) 5

d) >5

Correct Answer - A
Ans. A. 4.0
om
The vaginal acidity is due to lactic acid.
The normal pH in the healthy women of the child bearing age group
r.c

is 4.5
ke
an
tR
irs
.F
w
w
w

www.FirstRanker.com
www.FirstRanker.com www.FirstRanker.com

230. Vaginal pH before puberty is?


a) 7

b) 6

c) 4.5

d) 5

Correct Answer - A
Ans. A. 7 om
r.c
ke
an
tR
irs
.F
w
w
w

www.FirstRanker.com
www.FirstRanker.com www.FirstRanker.com

231. Decidual reaction is due to which


hormone ?
a) Progesterone

b) Estrogen

c) LH

d) FSH

Correct Answer - A
Ans. A. Progesterone
om
Increased structural & secretory activity of the endometrium that
brought about in response to progesterone following implantation is
r.c

known as Decidual reaction".


ke
an
tR
irs
.F
w
w
w

www.FirstRanker.com
www.FirstRanker.com www.FirstRanker.com

232. Inhibin levels are checked on which day


of menstrual cycle ?
a) Day 3

b) Day 4

c) Day 5

d) Day 6

Correct Answer - A
Ans. A. Day 3
om
Ovarian reserve test is designed to assess both the number of
immature eggs in the ovaries and their quality, which gives an
r.c

indication of woman's potential fertility.


ke

It works by detecting the levels of three female hormones,


an

using a blood sample taken on day three of menstrual cycle:


tR

1. Follicle stimulating hormone (FSH)


irs

2. Anti-mullerian hormone (AMH)


.F

3. Inhibin-B
w
w
w

www.FirstRanker.com
www.FirstRanker.com www.FirstRanker.com

233. Best test for ovulation ?


a) Serum estrogen

b) Serum progesterone

c) Both

d) None

Correct Answer - B
Ans. B. Serum progesterone
Plasma concentration of progesterone rises after ovulation &
reaches peak of 15 ng/ml at mid luteal phase & then declines as the
om

corpus luteum degenerates".


r.c
ke
an
tR
irs
.F
w
w
w

www.FirstRanker.com
www.FirstRanker.com www.FirstRanker.com

234. Cardiac output increases maximum at


which week?
a) 26-28wks

b) 30-32 wks

c) 32-34 wks

d) 34-36 wks

Correct Answer - C
Ans. C. 32-34 wks
om
Cardiac output:
Starts to increase from 5th week of pregnancy, reaches its peak 40-
r.c

50% at about 30-34 weeks.


ke
an
tR
irs
.F
w
w
w

www.FirstRanker.com
www.FirstRanker.com www.FirstRanker.com

235. In pregnancy plasma volume increased


maximum at what gestational age?
a) 10 wks

b) 20 wks

c) 25 wks

d) 30 wks

Correct Answer - D
Ans, D. 30 wks
om
Plasma volume is increased. Starting to increase at 6 weeks &
reaching max upto 50% at 30 weeks. Total plasma volume
r.c

increases to the extent of 1.25 litres.


ke
an
tR
irs
.F
w
w
w

www.FirstRanker.com
www.FirstRanker.com www.FirstRanker.com

236. 5 month pregnant female, which of the


following is true?
a) 50% have soft systolic murmur

b) Cardiac output is reduced

c) Systemic vascular resistance is increased

d) Increase in CVP

Correct Answer - A
Ans. A. 50% have soft systolic murmur
om
Anatomical changes during pregnancy
Heart is pushed upwards & outward. Apex beat is shifted in 4th
r.c

intercostals space.
ke

A systolic murmur can be heard in apical or pulmonary area.


an

Mammary murmur is a continuous hissing murmur audible over


tR

tricuspid area in left 2nd & 3rd iintercostals space.


irs

ECG shows left axis deviation. 53 and rarely 54 can be heard.


.F
w
w
w

www.FirstRanker.com
www.FirstRanker.com www.FirstRanker.com

237. Spinnbarkeit is maximum shown at


which phase?
a) Menstrual phase

b) Ovulatory

c) Post ovulatory

d) Pre follicular

Correct Answer - B
Ans. B. Ovulatory
om
Spinnbarkeit test (Thread test or Fern test)
A sPecimen of cervical mucus when seen under low power
r.c

microscope, shows a characteristic fern formation.


ke

Done to see the estrogenic activity in ovulatory phase of menstrual


an

cycle.
tR
irs
.F
w
w
w

www.FirstRanker.com
www.FirstRanker.com www.FirstRanker.com

238. Rarest presentation is?


a) Cephalic

b) Breech

c) Shoulder

d) Vertex

Correct Answer - C
Ans. C. Shoulder om
r.c
ke
an
tR
irs
.F
w
w
w

www.FirstRanker.com
www.FirstRanker.com www.FirstRanker.com

239. Fertile period of female is measured by ?


a) LH

b) FSH

c) Estrogen

d) Oxytocin

Correct Answer - A
Ans, A. LH
"LH surge from the anterior pituitary gland occurs 24 hours prior to
ovulation. Radioimmunoassays of the morning sample of urine &
om

blood gives results in 3 hours. Not only does the LH surge help in
r.c

predicting ovulation, but the approximate time of ovulation can be


ke

gauged & coitus around this time can improve the chances of
an

conception."
tR
irs
.F
w
w
w

www.FirstRanker.com
www.FirstRanker.com www.FirstRanker.com

240. External version is done after?


a) 34 weeks

b) 36 weeks

c) 38 weeks

d) 40 weeks

Correct Answer - B
Ans. B. 36 weeks
The maneuver is carried out after 36 weeks in labour-delivery
complex."
om
r.c
ke
an
tR
irs
.F
w
w
w

www.FirstRanker.com
www.FirstRanker.com www.FirstRanker.com

241. Least common presentation of twins ?


a) Both vertex

b) Both breech

c) Both transverse

d) First vertex and 2nd transverse

Correct Answer - C
Ans. C.Both transverse
Lie - Presentation in twin pregnancy:
The most common lie in the fetuses is longitudinal (90%).
om

The combination of presentation of fetuses are:


r.c

Both vertex (50%)


ke

First breech second vertex (10%)


an

First vertex and second transverse (rare)


tR

First yertex and second breech (30o/o)


irs

Both breech (10%)


.F

Both transverse (rarest).


w
w
w

www.FirstRanker.com
www.FirstRanker.com www.FirstRanker.com

242.

om
r.c
ke
an
tR
irs
.F
w
w
w

www.FirstRanker.com
www.FirstRanker.com www.FirstRanker.com

Presenting part in transverse lie ?


a) Shoulder

b) Face

c) Vertex

d) Brow

Correct Answer - A
Ans. A. Shoulder
'When the long axis of the fetus lies perpendicular o the maternal
spine or centralised uterine axis, it is called transverse lie.
om

But more commonly, fetal axis lies oblique to the maternal spine & is
r.c

then called oblique lie. In either of the conditions shoulder usually


ke

presents over the cervical opening during labour & both are
an

collectively called shoulder presentations."


tR
irs
.F
w
w
w

www.FirstRanker.com
www.FirstRanker.com www.FirstRanker.com

243. Severely anaemic pregnant patient in


cardiac failure. Choice of transfusion?
a) Platelets

b) Packed cells

c) Whole blood

d) Exchange transfusion

Correct Answer - B
Ans. B. Packed cells
om
r.c
ke
an
tR
irs
.F
w
w
w

www.FirstRanker.com
www.FirstRanker.com www.FirstRanker.com

244. Hydrops fetalis is due to ?


a) Rh mismatch

b) Hyperproteinemia

c) Placental hypoplasia

d) All of the above

Correct Answer - A
Ans. A. Rh mismatch om
r.c
ke
an
tR
irs
.F
w
w
w

www.FirstRanker.com
www.FirstRanker.com www.FirstRanker.com

245. Caput succedaneum indicates that fetus


was alive till ?
a) Immediately after birth

b) Till 2-3 days after birth

c) 2-3 weeks after birth

d) 2-3 months after birth

Correct Answer - A
Ans. A. Immediately after birth
om
CaPut succedaneum
Formation of swelling due to stagnation of fluid in the layers of scalp
r.c

beneath the girdle of contact.


ke

Boggy diffuse swelling, not limited to midline.


an

Disappears spontaneously within 24 hours.


tR

Usually occurs after rupture of membranes.


irs
.F
w
w
w

www.FirstRanker.com
www.FirstRanker.com www.FirstRanker.com

246. Caput succedeum is said to occur in


baby?
a) Within 24 hrs

b) 2-3 days

c) 2-3 weeks

d) 2-3 months

Correct Answer - A
Ans. A. Within 24 hrs
om
r.c
ke
an
tR
irs
.F
w
w
w

www.FirstRanker.com
www.FirstRanker.com www.FirstRanker.com

247. For uterine prolapse in pregnancy, Ring


pessary can be inserted upto ?
a) 12 weeks

b) 14 weeks

c) 16weeks

d) 18 weeks

Correct Answer - D
Ans. D. 18 weeks
om
Pessary treatment for ProlaPse
It does not cure the prolapse, only gives symptomatic relief by
r.c

stretching the hiatus genitalis, thus preventing the uterine & vaginal
ke

descent.
an

Indications of PessarY
tR

EarlY Pregnancy - uPto 18 weeks


irs

PuerPerium
.F

Patients absolutely unfit for surgery


w

Patients unwilling for operation


w
w

While waiting for surgery


Additional benefits like improvement of urinary symptoms

www.FirstRanker.com
www.FirstRanker.com www.FirstRanker.com

248. Not a method for delivery of after-coming


head of breech ?
a) Forceps method

b) Burns and Marshall method

c) Malar flexion and shoulder traction

d) Half hand method

Correct Answer - D
Ans. D. Half hand method
om
Methods of delivery of aftercoming head are
Burns - Marshall method
r.c

ForcePs delivery
ke

Malar flexion and shoulder traction (modified Mauriceau-Smellie-Veit


an

technique)
tR
irs
.F
w
w
w

www.FirstRanker.com
www.FirstRanker.com www.FirstRanker.com

249. Investigation of choice in post


menopausal bleeding?
a) PAP smear

b) Laproscopy

c) Fractional curettage

d) Ultrasound

Correct Answer - C
Ans., C. Fractional curettage
om
r.c
ke
an
tR
irs
.F
w
w
w

www.FirstRanker.com
www.FirstRanker.com www.FirstRanker.com

250. Preferred IUD for menorrhagea ?


a) NOVA T

b) Cu IUD

c) Mirena

d) Gyne fix

Correct Answer - C
Ans. C. Mirena om
r.c
ke
an
tR
irs
.F
w
w
w

www.FirstRanker.com
www.FirstRanker.com www.FirstRanker.com

251. Preferred treatment for menorrhagea in


reproductive age group?
a) NOVA T

b) Cu IUD

c) OCPs

d) Hysterectomy

Correct Answer - C
Ans. C. OCPs
om
r.c
ke
an
tR
irs
.F
w
w
w

www.FirstRanker.com
www.FirstRanker.com www.FirstRanker.com

252. Drug not used commonly for


menorrhagea ?
a) Methergin

b) Clomiphene

c) GnRH

d) NSAIDS

Correct Answer - A
Ans. A. Methergin
om
r.c
ke
an
tR
irs
.F
w
w
w

www.FirstRanker.com
www.FirstRanker.com www.FirstRanker.com

253. Drug causing abruption placenta ?


a) Methadone

b) Cocaine

c) Amphetamine

d) Fluoxetine

Correct Answer - B
Ans. B. Cocaine om
r.c
ke
an
tR
irs
.F
w
w
w

www.FirstRanker.com
www.FirstRanker.com www.FirstRanker.com

254.

om
r.c
ke
an
tR
irs
.F
w
w
w

www.FirstRanker.com
www.FirstRanker.com www.FirstRanker.com

Classical C section indicated in ?


a) CA Cervix

b) Central placenta praevia

c) Failed induction

d) Fetal distress

Correct Answer - A
Ans. A. CA Cervix om
r.c
ke
an
tR
irs
.F
w
w
w

www.FirstRanker.com
www.FirstRanker.com www.FirstRanker.com

255. Definitive indication of LSCS ?


a) Mento ant

b) Contracted pelvis

c) Occipito posterior

d) Vertex

Correct Answer - B
Ans, B. Contracted pelvis om
r.c
ke
an
tR
irs
.F
w
w
w

www.FirstRanker.com
www.FirstRanker.com www.FirstRanker.com

256. After delivery upto which week is known


as puerperium?
a) 2 weeks

b) 4 weeks

c) 6 weeks

d) 8 weeks

Correct Answer - C
Ans. C. 6 weeks
om
Immediate puerperium: 24 h
Early puerperium: upto 1 week
r.c

Remote puerperium: upto 6 weeks


ke
an
tR
irs
.F
w
w
w

www.FirstRanker.com
www.FirstRanker.com www.FirstRanker.com

257. Which of these steps is followed first for


the management of shoulder dystocia
after McRoberts maneuver?
a) Sharp flexion of hip joints towards abdomen

b) Supra pubic pressure

c) 90 degree rotation of posterior shoulder

d) Emergency c-section

Correct Answer - B
om

Ans. B.Supra pubic pressure


r.c

Applying a gentle suprapubic pressure is the first step in


management of shoulder dystocia after McRoberts maneuver.
ke

Mnemonic “HELPERR” as a guide for treating shoulder


an

dystocia:
tR

“H” stands for help


irs

“E” stands for evaluate for episiotomy.


.F

“L” stands for legs(pull your legs toward your stomach McRoberts
w
w

maneuver)
w

“P” stands for suprapubic pressure


“E” stands for enter maneuvers(internal rotation of baby’s shoulders)
“R” stands for remove the posterior arm from the birth canal.
“R” stands for roll the patient.

www.FirstRanker.com
www.FirstRanker.com www.FirstRanker.com

258. Peripartum cardiomyopathy occurs at-


a) Within 7 days

b) Within 6 weeks

c) Within 24 months

d) Within 5 months

Correct Answer - D
Ans. D. Within 5 months
PPCM is a structural heart muscle disease that occurs in women
either at the end of pregnancy or up to five months after giving birth.
om

Pearson’s diagnostic criteria for peripartum cardiomyopathy:


r.c

Development of cardiac failure in the last month of pregnancy or


ke

within 5 months after delivery


an

Absence of an identifiable cause for the cardiac failure


tR

Absence of recognizable heart disease prior to the last month of


irs

pregnancy, and
.F

Left ventricular systolic dysfunction demonstrated by classic


w

echocardiographic criteria, such as depressed ejection fraction or


w

fractional shortening along with a dilated left ventricle.


w

www.FirstRanker.com
www.FirstRanker.com www.FirstRanker.com

259. Nerve mostly compressed in pregnancy


puerperium:
a) Radial nerve

b) Median nerve

c) Femoral nerve

d) Facial nerve

Correct Answer - C
Ans C. Femoral nerve
om
The most common postpartum nerve injury is foot drop due to injury
to the peroneal nerve and injury to the lateral femoral cutaneous
r.c

nerve.
ke

Obstetricians may consider frequent position changes in labor,


an

avoidance of prolonged hip flexion, and shortening the pushing time


tR

by allowing for passive descent of the fetus before pushing begins


irs

as means of avoiding lateral femoral cutaneous nerve injury.


.F
w
w
w

www.FirstRanker.com
www.FirstRanker.com www.FirstRanker.com

260. In pregnancy which of the following level


is altered mostly:
a) Total T3

b) Free T3

c) Free T4

d) TSH

Correct Answer - A
Ans A. Total T3
om
Total T3 levels are altered throughout pregnancy.
Hormones during pregnancy:
r.c

Increased Decreased Unchanged


ke

Growth Hormone Luteinizing hormone ADH


an

Follicle-stimulating Free
tR

hormone T3,Free T4
irs

Di-Hydro
.F

Prolactin
EpiAndroSterone
w
w

Total T3, Total T4


w

Aldosterone
Testosterone,Androstenodione
and cortisol
Insulin(due to insulin resistance)
Estrogen
Progesterone
Total T3 and total T4 increase due to stimulation of thyroid tissue by
hCG during the early period of gestation. But there is no change in
free T3 and T4 due to increase in TBG.
This leads to transient fall in TSH during an early weeks of gestation.

www.FirstRanker.com
www.FirstRanker.com www.FirstRanker.com

The level of TSH reaches normal level during second and third
trimester.

om
r.c
ke
an
tR
irs
.F
w
w
w

www.FirstRanker.com
www.FirstRanker.com www.FirstRanker.com

261. What is meant by Superfecundation?


a) Fertilization of two or more ova in one intercourse

b) Fertilization of two or more ova in different intercourses in same


menstrual cycle

c) Fertilization of ova and then it's division

d) Fertilization of second ovum first being implanted

Correct Answer - B
Ans. B. Fertilization of two or more ova in different intercourses
in same menstrual cycle
om

Superfecundation is the fertilization of two or more ova from the


r.c

same cycle by sperm from separate acts of sexual intercourse,


ke

which can lead to twin babies from two separate biological fathers.
an

The term superfecundation is derived from fecund, meaning the


tR

ability to produce offspring.


irs
.F
w
w
w

www.FirstRanker.com
www.FirstRanker.com www.FirstRanker.com

262. Acute fatty liver commonly seen in


pregnancy at-
a) 3rd trimester

b) 1st trimester

c) Immediate postpartum

d) Intrapartum

Correct Answer - A
Ans. A. 3rd trimester
om
Acute fatty liver of pregnancy (AFLP) is a rare, potentially fatal
complication that occurs in the third trimester or early postpartum
r.c

period.
ke

Acute fatty liver of pregnancy is more commonly associated with:


an

Male fetus
tR

First pregnancy
irs

Maternal obesity
.F

It is due to defect in the long-chain-3-hydroxyacyl-CoA-


w

dehydrogenase pathway.
w
w

It is uncommon in subsequent pregnancies.

www.FirstRanker.com
www.FirstRanker.com www.FirstRanker.com

263. Fetal heart starts contracting at-


a) 10-12 days

b) 10-12 weeks

c) 3-5 weeks

d) 3- 5 month

Correct Answer - C
Ans. C 3-5 weeks
The fetal heart starts contracting at approximately 23 days of
gestation.
om
r.c
ke
an
tR
irs
.F
w
w
w

www.FirstRanker.com
www.FirstRanker.com www.FirstRanker.com

264. Which of the following is correct


regarding placenta?
a) Placental artery provides nutrients through umbilical cord to
baby

b) Placenta has Wharton's jelly

c) Placenta has 2 veins and 1 artery

d) Estrogen is secreted by placenta

Correct Answer - B
om
Ans. B. Placenta has Wharton's jelly
The umbilical cord is a structure that provides vascular flow between
r.c

the fetus and the placenta.


ke

It contains two arteries and one vein, which are surrounded and
an

supported by gelatinous tissue known as Wharton's jelly.


tR
irs
.F
w
w
w

www.FirstRanker.com
www.FirstRanker.com www.FirstRanker.com

265.

om
r.c
ke
an
tR
irs
.F
w
w
w

www.FirstRanker.com
www.FirstRanker.com www.FirstRanker.com

Anesthesia of choice for cesarean section in


severe pre-eclampsia :
a) Spinal

b) GA

c) Epidural

d) Spinal+epidural

Correct Answer - C
Ans. C. Epidural
om
Continuous Epidural Anesthesia is the first choice for patients with
preeclampsia during labour, Vaginal delivery and cesarean section.
r.c

Preeclampsia patients have a risk of severe airway edema , which


ke

makes intubation difficult


an

Continuous Epidural Anesthesia can improve uteroplacental


tR

perfusion and also decrease catecholamine secretions.


irs
.F
w
w
w

www.FirstRanker.com
www.FirstRanker.com www.FirstRanker.com

266. Which of the following is not a high-risk


pregnancy?
a) Previous history of manual removal of placenta

b) Anemia

c) Diabetes

d) Obesity

Correct Answer - A
Ans. A. Previous history of manual removal of placenta
om
High Risk Pregnancy is seen in :
Cardiovascular and renal factors:
r.c

Moderate to severe preeclampsia


ke

Chronic hypertension
an

Severe heart failure (class II-IV, NYHA classification)


tR

Moderate to severe renal disorders


irs

Metabolic disorders:
.F

Obesity
w

Insulin-dependent diabetes
w
w

Previous endocrine ablation


Obstetric history:
Fetal exchange transfusion because of Rh incompatibility
Stillbirth
Late abortion (16–20 wk)
Post-term pregnancy (> 42 wk)
Preterm newborn (< 37 wk and < 2500 g)
Intrauterine growth restriction (weight < 10th percentile for estimated
gestational age)
Polyhydramnios
Multifetal pregnancy

www.FirstRanker.com
www.FirstRanker.com www.FirstRanker.com

Previous brachial plexus injury


Abnormal fetal position
Other disorders:
Abnormal cervical cytologic findings
Sickle cell disease
Severe anemia
Thrombophilia
Autoimmune disorders
Anatomic abnormalities:
Uterine malformation
Insufficient (incompetent) cervix
Exposure to teratogens:
Group B streptococcal infections
Smoking > 10 cigarettes/day (associated with premature rupture of
membranes)
Current pregnancy complications:
Preterm labor at < 37 Wks
Preterm premature rupture of membranes
om

Moderate to severe preeclampsia


r.c

Polyhydramnios (hydramnios) or oligohydramnios


ke

Post-term (> 42 wk)


an

Uterine rupture
tR

Placenta previa
irs

Abruptio placentae
Chorioamnionitis
.F
w

Prolapsed cord
w

Fetal bradycardia for >30 min


w

Fetal tachycardia for >30 min


Fetal weight < 2.5 kg
Fetal weight > 4 kg
Fetal acidosis pH ≤7

www.FirstRanker.com
www.FirstRanker.com www.FirstRanker.com

267. Which of the following is not used in


preeclampsia?
a) Methyldopa

b) Atenolol

c) Labetalol

d) Hydralazine

Correct Answer - B
Ans. B. Atenolol
om
Choice of anti-hypertensives in pregnancy
PIH / pre-eclampsia / severe pre-eclampsia : Labetalol
r.c

Acute Hypertension/Hypertension crisis : IV labetalol > Nifedipine


ke

> Hydralazine
an

Chronic Hypertension in pregnancy : Labetalol > Methyl dopa.


tR
irs
.F
w
w
w

www.FirstRanker.com
www.FirstRanker.com www.FirstRanker.com

268. Postmenopausal women 1st line of drug


for osteoporosis is-

a) OCP

b) Bisphosphonates

c) Raloxifene

d) Strontium

Correct Answer - B
om

Ans. is. B. Bisphosphonates


r.c

First line of management of osteoporosis – Bisphosphonates


females.
ke

Alendronate, etidronate, pamidronate, and Ibandronate are


an

bisphosphonates which inhibit bone resorption, and are very


tR

effective for both osteoporosis prevention and treatment.


irs

Uses:
.F

First line drugs for treating postmenopausal osteoporosis


w
w

Paget’s disease
w

Osteolytic bone metastasis.


Raloxifene: It is a selective estrogen receptor modulator which is
also useful in management of osteoporosis but it is a hormonal
preparation.

www.FirstRanker.com
www.FirstRanker.com www.FirstRanker.com

269. A 36-week old pregnant lady with


previous twin delivery. What is the
Ovarian score?
a) G2P1

b) G2P2

c) G3P2

d) G3P3

Correct Answer - A
om

ANS. A.G2P1
r.c

Gravida includes all confirmed pregnancies. Each pregnancy is only


counted one time, even if the pregnancy was multiple gestations,
ke

such as twins or triplets


an

Parity reflects the total number of births after 20 weeks, not the total
tR

number of infants born.


irs

Ref.OBS Dutta 8th ed Page no. 107


.F
w
w
w

www.FirstRanker.com
www.FirstRanker.com www.FirstRanker.com

270. DCDA twins, 38wks, first twin breech


mother has BP 140/96, 1+ proteinuria
what's the management
a) Immediate LSCS

b) Induction at 40 weeks

c) Immediate induction and delivery

d) Induction if signs of preeclampsia

Correct Answer - A
om

ANS. A. Immediate LSCS


r.c

CASE DISCUSSION:
The patient has mild preeclampsia with BP 140 / 96, so TOP is 37
ke

weeks.
an

This patient is 38 weeks so immediate termination of pregnancy is


tR

required.
irs

The first twin is breech so the mode of delivery is LSCS hence the
.F

answer is A.
w
w

INDICATIONS OF CESAREAN SECTION:


w

The indications are broadly divided into:


Obstetric
Placenta previa
Preeclampsia
Previous cesarean section
Cord prolapse of the first baby
Abnormal uterine contractions
Contracted pelvis.
For twins:
Both the fetuses or even the first fetus with noncephalic (breech or

www.FirstRanker.com
www.FirstRanker.com www.FirstRanker.com

transverse) presentation
Twins with complications: IUGR, conjoined twins
Monoamniotic twins
Monochorionic twins with TTTS
The collision of both the heads at brim preventing engagement of
either head.
Ref.OBS Dutta 8th ed Page no. 244

om
r.c
ke
an
tR
irs
.F
w
w
w

www.FirstRanker.com
www.FirstRanker.com www.FirstRanker.com

271. A 32-year-old woman complains of


amenorrhea since the delivery of a baby
15 months previously, despite the fact
that she did not breastfeed her baby. The
delivery was complicated by excessive
hemorrhage that required a transfusion
of 2.5 liters of blood. She has also been
fatigued and has gained an additional
4.5Kg since the baby was born.
Laboratory data show the following:
Serum LH < 1 IU/L (normal, 4-24 IU/L)
Serum estradiol 5 pg/mL (normal, 20 -
100 pg/mL)
om

Serum TSH 0.1 mU/L (normal, 0.5 - 5


r.c
ke

mU/L)
an

Serum GH 3 ng/mL (normal, < 5 ng/mL)


tR

Serum ACTH 28 pg/mL (normal, 10 - 50


irs
.F

pg/mL)
w
w

Serum prolactin 2 ng/mL (normal,


w

Injection of 500 μg of TRH failed to


produce the expected rise in both serum
TSH and prolactin. Which of the
following diagnoses most likely explains
the findings in this patient?
a) Hashimoto's thyroiditis

b) Isolated gonadotropin deficiency

www.FirstRanker.com
www.FirstRanker.com www.FirstRanker.com

b) Isolated gonadotropin deficiency

c) Primary amenorrhea

d) Sheehan's syndrome

Correct Answer - D
Ans. D. Sheehan's syndrome
Sheehan's syndrome
It is hypopituitarism due to ischemic damage to the pituitary resulting
from excessive hemorrhage during parturition.
The pituitary is enlarged during pregnancy; it is more metabolically
active, and more susceptible to hypoxemia.
Furthermore, the blood vessels in the pituitary may be more
susceptible to vasospasm because of the high estrogen.
In about 30% of women who hemorrhage excessively during
om

parturition, some degree of hypopituitarism eventually becomes


r.c

manifest.
ke

The symptoms depend on how much of the pituitary is damaged and


an

what cell types are destroyed. The patient described above exhibited
tR

persistent amenorrhea after the delivery of her infant.


irs

This is due to the destruction of pituitary gonadotrophs and


.F

diminished secretion of gonadotropins (LH).


w

There also appears to have been significant destruction of


w

lactotrophs since TRH injection failed to induce an increase in


w

prolactin.
Had the women attempted to breastfeed her infant, a failure to
lactate mostly likely would have occurred.
This case is also characterized by secondary hypothyroidism.
The low TSH and failure to respond to TRH injection are
confirmatory.
Corticotrophs appear to have been spared since plasma ACTH is
normal.
It is not clear whether somatotrophs were damaged.
Further testing would be needed to see if GH reserve is diminished.

www.FirstRanker.com
www.FirstRanker.com www.FirstRanker.com

Ref. William Obs. ed. 24th page no.798, 1163


Dutta Gynaecology 6th ed. Page no. 465

om
r.c
ke
an
tR
irs
.F
w
w
w

www.FirstRanker.com
www.FirstRanker.com www.FirstRanker.com

272. A 7 weeks pregnant lady has 1 accidental


exposure to x-ray. Which of the following
should be done?
a) Continue pregnancy

b) Terminate pregnancy

c) Chromosome analysis

d) Pre invasive diagnostic testing

Correct Answer - A
om

Ans. A. Continue pregnancy


r.c

Exposure to X-ray has a very low range of rad so in given case


scenario Pregnancy will be continued.
ke

Exposure >15 rad during the second and third trimester or >5 rad in
an

the first trimester needs patient counseling. Elective termination of


tR

pregnancy may be considered.


irs

Ref. OBS Dutta 8th edition Page no. 740


.F

William’s Obs 24th edition Page no. 931 Table no 46-5


w
w
w

www.FirstRanker.com
www.FirstRanker.com www.FirstRanker.com

273. In Modern obstetrics, for sensitized Rh-


negative mother what should be done to
evaluate the condition of the mother?
a) MCA doppler peak systolic volume

b) Fetal blood

c) Amniocentesis

d) Biophysical profile

Correct Answer - A
om

Ans. A. MCA doppler peak systolic volume


r.c

Doppler ultrasound: Serial Doppler study of the middle cerebral


artery (MCA)-peak systolic velocity (PSV) is the mainstay to assess
ke

fetal anemia. A value >1.5 multiples of the median (MOMs) for the
an

corresponding gestational age, predicts moderate to severe fetal


tR

anemia.
irs

This value (between 24 weeks and 35 weeks of gestation), is an


.F

indication for cordocentesis and fetal transfusion


w
w

Most centers have replaced serial amniocentesis with serial MCA


w

Doppler studies.
Ref. OBstetrics Dutta ed. 8th pg. 391

www.FirstRanker.com
www.FirstRanker.com www.FirstRanker.com

274. A patient delivered at home with a


complete perineal tear came to the
hospital after 2 weeks. What
management will you prefer?
a) Immediate repair

b) Repair 3 weeks post-delivery

c) Repair 6 weeks post-delivery

d) Repair 3 months post-delivery


om

Correct Answer - D
r.c

Ans. D. Repair 3 months post-delivery


ke

The recent tears should be repaired immediately following the


an

delivery of the placenta. This reduces the chance of infection and


tR

minimizes blood loss.


irs

In cases of delay beyond 24 hours, the repair is to be withheld.


.F

Antibiotics should be started to prevent infection.


w

The complete tear should be repaired after 3 months if delayed


w

beyond 24 hours.
w

In case of any doubt to the grade 3rd-degree tear, it is advisable to


classify to a higher degree rather than a lower degree.
Ref.Obs Dutta 24th edition Page no. 490

www.FirstRanker.com
www.FirstRanker.com www.FirstRanker.com

275. All are used for postcoital contraception


except-
a) CuT

b) Ru 486

c) High dose estrogen

d) Danazol

Correct Answer - D
Ans. D. Danazol
om
EMERGENCY CONTRACEPTIVES:
Drug Dose PREGNANCY
r.c

RATE
ke

Levonorgestrel 0.75 mg stat and 0–1


an

after 12 hours
tR

Ethinyl estradiol 50 µg + 2 tab stat and 2 after 0–2


irs

norgestrel 0.25 mg 12 hours


.F
w

Ethinyl estradiol 2.5 mg Bd × 5 days 0–0.6


w

Mifepristone 100 mg single dose 0–0.6


w

Copper Iuds (Gold standard) Insertion within 5 0–0.1


days
(RU 486Mifepristone) 30 mg PO 0–1
(ulipristal acetate)
Ref:DUTTA GYNAECOLOGY ED. 6TH PAGE NO. 492

www.FirstRanker.com
www.FirstRanker.com www.FirstRanker.com

276. 33 yr old female with heavy menstrual


bleeding for 6 months comes to the
gynaecology OPD. On examination, no
abnormality was seen. USG also
appeared normal. The patient was tried
to be managed on non-hormonal
treatment but it failed. What will be the
next management step?
a) Hormonal therapy

b) Endometrial sampling

c) Hysterectomy
om
r.c

d) Hysterectomy
ke
an
tR
irs

Correct Answer - B
.F

Ans. B.Endometrial sampling


w
w

Before starting Hormonal therapy endometrial sampling is done as if


w

hormonal therapy precedes sampling the sampling results will be


altered.
As already mentioned in question the USG and clinical examination
show no abnormality hence the diagnosis can’t be made on that so
for diagnosis we need to do endometrial sampling and see the
hormonal pattern. Then we can proceed with hormonal therapy.
Ref. Dutta Gynaecology ed. 6th page no. 120

www.FirstRanker.com

You might also like